Daily Self-Assessment Test Monthly Compilation September 2017 Compilation

GUIDED PREPARATION MODULE CSE PRELIMS 2018 48 Tests 24 Subject Sectional Tests 10 Current Affairs Tests 10 CSAT Tests 4 Comprehensive Tests

EXAM CENTRES: NEW DELHI & HYDERABAD ...... Office : Contact Us ; Subscribe at ; 18/15, Basement 011-45508600 www.jigyasaias.com Old Rajendra Nagar WhatsApp- 9650294824 Delhi - 110060 [email protected] 13/09/2017 Print Question Paper

1. Which of the following is/are appointed by the President of India:

1. Comptroller and Auditor General. 2. Chief Election Commissioner. 3. Union Minister.

Choose the correct option.

A. 1 only

B. 2 and 3 only

C. 1 and 2 only

D. All of the above

Correct Answer: D

Solution All of the are appointed by the President The CAG is appointed by the President Under article 148; The Chief Election Commissioner under article 324. and the Union Minister Under Article 75 Ref: M.Laxmikanth

2. Aggregate Measurement of Support (AMS) or ‗Amber Box‘ is a term often remains in the news, it is related to:

A. UN peace Forces

B. FDI in Manufacturing

C. WTO negotiations

D. Double Tax Avoidance Treaty

Correct Answer: C

Solution Recently (on 18 July 2017) India and China jointly submitted a proposal to the World Trade Organisation (WTO) calling for the elimination • by developed countries • of the most trade•distorting form of farm subsidies, known in WTO parlance as Aggregate Measurement of Support (AMS) or ‗Amber Box‘ support as a prerequisite for consideration of other reforms in domestic support negotiations. Ref: http://www.livemint.com/Politics/GSY4EQ3f09o1FoM5EB1FqI/India•China•jointly•oppose•tradedistorting•agricultural• su.html http://pib.nic.in/newsite/PrintRelease.aspx?relid=170392

3. The 'Rajaswa Gyan Sangam' is an annual conference aimed to:

A. GST registration facilitation

B. Enable a two•way communication between the policy•makers and the field officers

C. Ascertaining effects of Demonetisation on the economy

D. Tax Collection policies from unorganized sectors

Correct Answer: B

Solution The Central Board of Direct Taxes (CBDT) and the Central Board of Excise & Customs (CBEC) have been holding annual Conferences of senior officers for a number of years. In 2016, for the first time, a joint Conference of the two Boards was held under the umbrella of ―Rajaswa Gyan Sangam‖ which was inaugurated by the Hon‘ble Prime Minister. This year http://www.jigyasaias.com/Admin/printDailyTest/DTEST59A9287BEC720 1/4 13/09/2017 Print Question Paper

also, a joint Conference of the two Boards is scheduled to be held on 1st and 2nd of September, 2017

The objective of the Conference is to enable a two•way communication between the policy•makers and the senior officers in the field offices with a view to increase revenue collection and facilitate effective implementation of law and policies in key result areas. Apart from the joint sessions, separate Technical Sessions of each Board will be held during the two•day Conference. Issues arising in the implementation of policies and strategies to achieve targets in core functional areas will also be discussed. Such issues inter alia include HR issues, Litigation Management, Strategies for Revenue Maximisation, Tax Evasion, Taxpayer Services, GST and Customs Reforms, and Modernisation.

Ref: http://pib.nic.in/newsite/PrintRelease.aspx?relid=170413

4. Which of the following statements is/are correct about Comptroller and Auditor General of India:

1. He enjoys the security of tenure. 2. He is not eligible for further office either in Union or in State government. 3. No money could be withdrawal from the Consolidated Fund of India without the permission of CAG.

Choose the correct option.

A. 1 only

B. 2 and 3 only

C. 1 and 2 only

D. All of the above

Correct Answer: C

Solution Statement 1 and 2 are correct.

CAG is appointed by the President but he does not hold office under pleasure of President and can be removed only if Parliament impeaches him,

further, he cannot be appointed for any government post either in Union or in State government.

Statement 3 is however incorrect, there is no control of CAG at the stage of withdrawal of money, his role only comes in auditing stage.

Ref: M.Laxmikanth

5. Data related to estimates of Gross Domestic Product(GDP) is released by:

A. NITI Aayog

B. Central Statistics Office

C. Office of Economic Advisor

D. None of the above

Correct Answer: B

Solution The Central Statistics Office (CSO) under Ministry of Statistics and Programme Implementation releases the estimates of Gross Domestic Product(GDP).

Ref: http://pib.nic.in/newsite/printrelease.aspx?relid=170409

6. Which of the following industries is/are part of the 8 Core Industries of the economy?

1. Cement 2. Telecom 3. Fertilizers 4. Natural Gas. http://www.jigyasaias.com/Admin/printDailyTest/DTEST59A9287BEC720 2/4 13/09/2017 Print Question Paper

Choose the correct option.

A. 1, 2 and 3 only

B. 3 and 4 only

C. 1, 3 and 4 only

D. All of the above

Correct Answer: C

Solution Eight Core Industries are Electricity, Steel, Refinery products, Crude oil, Coal, Cement, Natural Gas, and Fertilizers. The Eight Core Industries comprise 40.27 % of the weight of items included in the Index of Industrial Production (IIP). Ref: http://pib.nic.in/newsite/printrelease.aspx?relid=170409

7. Swamp forests are classified as:

A. Forest cover at high altitude

B. Desert Forests

C. Temperate Evergreen forest

D. Wetland dominated by shrubs and trees

Correct Answer: D

Solution A swamp is a wetland that is dominated by woody plants (shrubs and trees). Swamps are often near rivers or streams. These rivers and streams sometimes flood and the water from the flood carries nutrients to the swamp.

Ref: http://ncert.nic.in/ncerts/l/kegy105.pdf

8. Which of the following statements is/are correct about Tuberculosis:

1. Tuberculosis is caused by a bacteria and most often affect the lungs. 2. It is a non•communicable disease 3. Tuberculosis is curable and preventable.

Choose the correct option.

A. 1 and 2 only

B. 2 and 3 only

C. 1 and 3 only

D. All of the above

Correct Answer: C

Solution Statement 1 and 3 are correct;

Tuberculosis (TB) is caused by bacteria (Mycobacterium tuberculosis) that most often affect the lungs. Tuberculosis is curable and preventable.

Statement 2 is incorrect;

TB is spread from person to person through the air. When people with lung TB cough, sneeze or spit, they propel the TB germs into the air. A person needs to inhale only a few of these germs to become infected.

Ref: http://www.who.int/mediacentre/factsheets/fs104/en/ http://www.jigyasaias.com/Admin/printDailyTest/DTEST59A9287BEC720 3/4 13/09/2017 Print Question Paper

9. Lonar lake is the largest crater lake in the country formed by a meteorite impact It is located in which state?

A.

B. Maharashtra

C. Karnataka

D. Goa

Correct Answer: B

Solution Lonar Lake is a notified National Geo•heritage Monument saline soda lake located at Lonar in Buldhana district, Maharashtra, India, which was created by a meteor impact during the Pleistocene Epoch and it is the only known hyper velocity impact crater in basaltic rock anywhere on Earth.

Ref: http://www.livemint.com/Science/6UwYpDDzbO8vMuUKN6er4K/Maharashtras•Lonar•Lake•area•is•shrinking• researchers•say.html

10. Consider the following statements about Polar Satellite Launch Vehicle (PSLV):

1. It is a single stage rocket. 2. It is capable of placing multiple payloads into orbit.

Which of the above is/are correct?

A. 1 only

B. 2 only

C. Both 1 and 2

D. Neither 1 nor 2

Correct Answer: B

Solution Statement 1 is incorrect, it is a 4 stage rocket. (Hardly any single stage rocket is used now.)

Statement 2 is correct.

The PSLV is capable of placing multiple payloads into orbit, thus multi•payload adaptors are used in the payload fairing.

Ref: http://www.isro.gov.in/launchers/pslv

http://www.thehindu.com/sci•tech/science/39•successes•later•pslv•launch•fails/article19596576.ece

http://www.jigyasaias.com/Admin/printDailyTest/DTEST59A9287BEC720 4/4 13/09/2017 Print Question Paper

1. Which of the following statements is/are correct about United Nations High Commissioner for Refugees (UNHCR)?

1. It was created during the aftermath of the Second World War, to help millions of Europeans who had fled or lost their homes. 2. It is the primary Refugee Agency of United Nations.

Choose the correct option.

A. 1 only

B. 2 only

C. Both 1 and 2

D. Neither 1 nor 2

Correct Answer: C

Solution Both the statements are correct.

The office of the United Nations High Commissioner for Refugees (UNHCR) was created in 1950, during the aftermath of the Second World War, to help millions of Europeans who had fled or lost their homes. We had three years to complete our work and then disband.

It is the primary UN agency for refugees.

Ref: http://www.unhcr.org.in/index.php?option=com_content&view=article&id=18&Itemid=103

2. Pradhan Mantri Matru Vandana Yojana provides for:

A. Widow Pension

B. Maternity Benefits

C. Old age pension for women

D. Free medical camps for old age women

Correct Answer: B

Solution The PMMVY is Centrally Sponsored Scheme under which the cost sharing ratio between the Centre and the States & UTs with Legislature is 60:40, for the North•Eastern States & three Himalayan States, it is 90:10 and 100% Central assistance for Union Territories without Legislature.

The draft guidelines inter•alia provide Aadhaar linkage, Direct Benefit Transfer of Rs. 5000 in beneficiary‘s bank/post office account in three instalments at the stage of early registration of pregnancy, after six months of pregnancy on at least one antenatal check•up and registration of child birth & first cycle of immunisation of the child.

Hence Option B is the most appropriate answer.

Ref: http://pib.nic.in/newsite/PrintRelease.aspx?relid=170448

3. The Ministry of Road Transport & Highways has recently rolled out FASTag programme, with the aim of:

A. Making National Highway free from Traffic Light

B. Repair and Maintainance of Highways

C. Fast Toll Collection

D. Speedy Clearance of pending Road Projects

Correct Answer: C

http://www.jigyasaias.com/Admin/printDailyTest/DTEST59AA8CE4E8B5E 1/4 13/09/2017 Print Question Paper

Solution Ministry of Road Transport & Highways has decided to roll out Electronic Toll Collection (ETC) program in the country under the brand name ―FASTag‖. ETC entails payment of highway tolls electronically without needing to stop the vehicle at the toll plazas. The unique number of the RFID Tag affixed on the wind shield of the vehicle will be read by the readers fitted in the dedicated ―ETC‖ lanes of plazas and appropriate amount as per the class of vehicle will be automatically deducted from the account of the user. The dedicated ETC lanes will have colour coding for distinct identity recognized as ―FASTag lanes‖.

Ref: PIB 1 sept 2017 Press Release

4. Fiscal Deficit is calculated as:

A. Total Expenditure (Subtracted by) Total Receipt.

B. Total Capital Expenditure (Subtracted by) Total Capital Receipt

C. Total Revenue Expenditure (Subtracted by) Total Revenue Receipt

D. Total Expenditure (Subtracted by) Non•Debt Creating Receipt

Correct Answer: D

Solution The gross fiscal deficit (GFD) is the excess of total expenditure including loans net of recovery over revenue receipts (including external grants) and non•debt capital receipts. The net fiscal deficit is the gross fiscal deficit less net lending of the Central government.

Generally fiscal deficit takes place either due to revenue deficit or a major hike in capital expenditure. Capital expenditure is incurred to create long•term assets such as factories, buildings and other development.

A deficit is usually financed through borrowing from either the central bank of the country or raising money from capital markets by issuing different instruments like treasury bills and bonds.

Ref: Ramesh Singh Economics book; http://economictimes.indiatimes.com/definition/fiscal•deficit

5. Which of the following statements is/are incorrect about Indian Ocean Rim Association (IORA):

1. It is a Military Block of countries bordering the Indian Ocean. 2. All sovereign States of the Indian Ocean Rim are eligible for membership of this Association. 3. Its Secretariat is located in New Delhi.

Choose the correct option

A. 1 and 2 only

B. 2 and 3 only

C. 1 and 3 only

D. All of the above

Correct Answer: C

Solution Statement 1 and 3 are incorrect;

It is not a military block, rather, IORA seeks to build and expand understanding and mutually beneficial cooperation through a consensus•based, evolutionary and non•intrusive approach. In keeping with this spirit, there are no laws, binding contracts or rigid institutional structures.

Cooperation is based on the principles of sovereign equality, territorial integrity, political independence, non•interference in internal affairs, peaceful coexistence and mutual benefit.

The Secretariat of the Indian Ocean Rim Association (IORA) is hosted by the Government of Mauritius and based in Cyber City, Ebène, Mauritius.

Statement 2 is however correct.

Ref: http://www.iora.net/charter.aspx

http://www.jigyasaias.com/Admin/printDailyTest/DTEST59AA8CE4E8B5E 2/4 13/09/2017 Print Question Paper

6. Which of the following countries are not members of BRICS?

A. Brazil

B. Canada

C. Russia

D. India

Correct Answer: B

Solution BRICS Members are:

Brazil; Russia; India; China; South Africa

Ref: http://brics2016.gov.in/content/

7. Which of the following can only be removed from his office by the President after Parliament passing an Adress with the absolute majority of its favor?

1. Comptroller and Auditor General of India. 2. Attorney General of India 3. Chief Election Commissioner 4. Union Cabinet Minister

Choose the correct option.

A. 1, 3 and 4 only

B. 2 and 3 only

C. 1 and 3 only

D. None of the Above

Correct Answer: C

Solution Chief Election Commissioner, Comptroller and Auditor General of India, Supreme Court Judge, High Court Judge can be removed in this manner.

While Attorney General and Union Cabinet minister hold office during the pleasure of the President and can be removed by him.

Ref: M.Laxmikanth

8. Which of the following states in India shares International Boundary with 3 Nations?

1. . 2. Assam 3. Jammu and Kashmir

Choose the correct option.

A. 1 and 3 only

B. 1 and 2 only

C. 2 and 3 only

D. All of the above

Correct Answer: A

http://www.jigyasaias.com/Admin/printDailyTest/DTEST59AA8CE4E8B5E 3/4 13/09/2017 Print Question Paper

Solution Sikkim share boundary with 3 countries Nepal, Bhutan, and China.

Jammu and Kashmir share boundary with China, Pakistan, and Afghanistan.

Assam Share boundary only with Bhutan and Bangladesh only.

Ref: Map

http://www.jigyasaias.com/Admin/printDailyTest/DTEST59AA8CE4E8B5E 4/4 13/09/2017 Print Question Paper

1. Which of the following statements is/are correct?

1. Only a sitting Member of Parliament can be appointed as a Cabinet Minister. 2. Union Ministers are administered the oath of office and secrecy by the Prime Minister. 3. Constitution provides for 3 classifications of Ministers in Union Government i.e Cabinet Ministers, Minister of State and Minister of State with Independent Charge.

Choose the correct option.

A. 3 only

B. 1 and 3 only

C. All of the above

D. None of the Above

Correct Answer: D

Solution All the statements are incorrect;

Statement 1 is incorrect as even a non•Member can be appointed as a Minister provided that he becomes a member in next 6 months.

Statement 2 is incorrect; Ministers are administered the oath of office and secrecy by the President (and not the PM). Statement 3 is also incorrect as; the Constitution provides no such classification it is developed as the convention. Ref: M.Laxmikanth

2. Which of the following is/are correct about Onam Festival?

1. It is celebrated by Telugu community in the states of Andhra Pradesh and Telangana. 2. It is a Harvest Festival which lasts for 10 days.

Choose the correct option

A. 1 only

B. 2 only

C. Both 1 and 2

D. Neither 1 nor 2

Correct Answer: B

Solution Statement 1 is incorrect, it is a harvest Festival of Kerala.

Onam is the biggest festival in the Indian state of Kerala. Onam Festival falls during the Malayali month of Chingam (Aug • Sep) and marks the homecoming of legendary King Mahabali. Carnival of Onam lasts for ten days and brings out the best of Kerala culture and tradition. Intricately decorated Pookalam, ambrosial Onasadya, breathtaking Snake Boat Race and exotic Kaikottikali dance are some of the most remarkable features of Onam • the harvest festival in Kerala.

Ref: http://indianexpress.com/photos/lifestyle•gallery/onam•2017•photos•celebrating•the•harvest•festival•of•kerala• 4826450/3/

3. Apart from 5 members states of BRICS 5 other nations are invited to attend the BRICS summit 2017 hosted by China.

Which of the following is not among the non•member invited?

A. Egypt

B. Indonesia

http://www.jigyasaias.com/Admin/printDailyTest/DTEST59AD15F2BDB48 1/4 13/09/2017 Print Question Paper

C. Kenya

D. Mexico Correct

Answer: B

Solution As you know Brazil, Russia, India, China, and South Africa are the members of BRICS, apart from these members, 5 Countries i.e. Egypt, Kenya, Tajikistan, Mexico, and Thailand are invited by China to attend the BRICS summit 2017 held at Xiamen City of China.

Ref: http://www.firstpost.com/world/china•invites•five•nations•for•brics•summit•denies•attempts•at•expanding•group• 3989949.html

4. What does the Heat Sheild in the PSLV rocket do?

A. It prevents circuits of the satellites from freezing in the open space.

B. It keeps the battery of satellite running when the satellite cannot generate energy from sun light

C. It protects the Satellite from the intense heat generated due to friction with the atmosphere during take off.

D. It protects satellite from the fire generated by the rocket.

Correct Answer: C

Solution Located in the fourth stage of the rocket, the heat shield, also referred to as payload fairing, protects the satellite from the intense heat generated due to friction with the atmosphere during take off.

The PSLV• C39 mission, carrying the IRNSS•1H navigation satellite, on Thursday failed after the heat shield refused to open and release the satellite.

Ref: http://www.thehindu.com/sci•tech/science/isro•scanning•data•on•pslv•c39•failure/article19611531.ece

5. Which of the following statements is/are correct about United Nation Security Council?

1. The Council is Composed of 15 members out of which 5 are permanent and 10 are non•permanent members. 2. USA, Russia, China, Japan and the United Kingdom are 5 permanent members of the council. 3. Non•Permanent members are elected for two•year terms by the General Assembly.

Choose the correct option.

A. 1 and 2 only

B. 2 and 3 only

C. 1 and 3 only

D. All of the above

Correct Answer: C

Solution The Second Statement is incorrect, 5 permanent Members are China, France, Russian Federation, the United Kingdom, and the United States. Japan is not a member.

Statement 1 and 3 are correct there are total 5 Permanent members and 10 non•permanent members.

And ten non•permanent members elected for two•year terms by the General Assembly (with the end of term date):

Ref: http://www.un.org/en/sc/members/

6. Tholpava Koothu, a famous art form of Kerala is a:

http://www.jigyasaias.com/Admin/printDailyTest/DTEST59AD15F2BDB48 2/4 13/09/2017 Print Question Paper

A. Shadow Puppet Play

B. Sanskrit Theatre

C. Folk Painting

D. Ancient Martial Art

Correct Answer: A

Solution Tholpava Koothu, the unique shadow puppet theatre form of Kerala, South India, is preserved as a family tradition by the Pulavars. It is an ancient peculiar ritualistic art form.

Ref: http://indianexpress.com/article/lifestyle/art•and•culture/tholpavakoothu•and•kamishibai•two•indigenous•storytelling• forms•4374610/ The Hindu 4 Sept 2017 Edition.

7. Demand is likely to be Inelastic for which of the following goods.

1. If it is a Necessity good. 2. If no substitutes of the good exist. 3. If its use can be postponed

Choose the correct option.

A. 1 only

B. 2 and 3 only

C. 1 and 2 only

D. All of the above

Correct Answer: C

Solution Inelastic means that when the price goes up, consumers‘ buying habits stay about the same, and when the price goes down, consumers‘ buying habits also remain unchanged. If the use of a product can be postponed it is likely that the consumers refrain from buying it if price rises and will wait for its fall and in case of fall in price will buy more of it.

ref: http://ncert.nic.in/ncerts/l/leec202.pdf

8. Which of the following statements is/are correct about Stubble Burning?

1. It is referred to the deliberate setting fire of the straw stubble that remains after wheat and other grains have been harvested. 2. It can cause an increase in air pollution.

Choose the correct option.

A. 1 only

B. 2 only

C. Both 1 and 2

D. Neither 1 nor 2

Correct Answer: C

Solution

http://www.jigyasaias.com/Admin/printDailyTest/DTEST59AD15F2BDB48 3/4 13/09/2017 Print Question Paper

Stubble burning is the deliberate setting fire of the straw stubble that remains after wheat and other grains have been harvested. Governments increasingly restricted its use.

Though it is beneficial for the farmers in short run, but it causes a lot of air pollution because of which NGT has banned this practice.

Ref: http://ptinews.com/news/9036183_Stubble•burning••NGT•asks•4•states•to•show•cases•of•farmers•given•incentive

9. FARC is a Marxist Guerilla Organisation often remains in the news. It is related to which country?

A. Chile

B. Comoros

C. Cambodia

D. Columbia

Correct Answer: D

Solution The Farc (Fuerzas Armadas Revolucionarias de Colombia) is the oldest and largest group of Colombia's left•wing rebels and is one of the world's richest guerrilla armies.

After half of century of fighting in the jungles and mountains of Colombia, the FARC, announced its transformation into a political party this week, vowing to respect the rules of democracy while sticking to its left•wing principles.

Ref: https://www.ft.com/content/df7f2840•8f79•11e7•a352•e46f43c5825d

10. Sardar Sarovar Dam on river Narmada is located in which state?

A. Madhya Pradesh

B. Gujrat

C. Rajasthan

D. Maharashtra

Correct Answer: B

Solution The Sardar Sarovar Project is one of the largest water resources project of India covering four major states • Maharashtra, Madhya Pradesh, Gujarat, and Rajasthan. Dam's spillway discharging capacity (30.7 lakhs cusecs) would be third highest in the world.

With 1133 cumecs (40000 cusecs) capacity at the head regulator, and 532 km. length, the Narmada Main Canal would be the largest irrigation canal in the world.

Though the project will be helpful to many states the Dam is located in the state of Gujrat.

Ref: http://www.firstpost.com/india/sardar•sarovar•dam•people•staying•put•in•submergence•area•despite•rise•in•water• level•says•official•3999641.html

http://www.jigyasaias.com/Admin/printDailyTest/DTEST59AD15F2BDB48 4/4 13/09/2017 Print Question Paper

1. In which of the following International Organisations, India and both are full members?

1. SAARC 2. East Asia Summit. 3. ASEAN 4. BIMSTEC.

Choose the correct option.

A. 1, 3 and 4 only

B. 2 and 4 only

C. 1, 2 and 3 only

D. All of the above

Correct Answer: B

Solution SAARC members are Afghanistan, Bangladesh, Bhutan, India, Nepal, the Maldives, Pakistan and Sri Lanka. Hence Myanmar is not a member.

ASEAN members are Indonesia, Malaysia, Philippines, Singapore, Thailand, Brunei, Vietnam, Laos, Myanmar, Cambodia. India is not a full member of ASEAN.

In East Asia Summit Both the countries are members along with, Brunei Darussalam, Cambodia, Indonesia, Lao PDR, Malaysia, Myanmar, Singapore, Thailand, the Philippines and Vietnam, Australia, China, India, Japan, New Zealand, Republic of Korea, Russian Federation and the USA.

In BIMSTEC also both the nations are members among Bangladesh, India, Myanmar, Sri Lanka, Thailand, Bhutan, and Nepal.

Ref: MEA website.

2. 'Hortinet' is an integrated system being developed for providing Internet based electronic services for facilitating: testing, certification, monitoring, traceability etc for horticultural produce in India.

This system is developed by:

A. Council of Scientific & Industrial Research (CSIR)

B. NITI Aayog

C. Ministry of Consumer Affairs

D. Agricultural and Processed Food Products Export Development Authority (APEDA)

Correct Answer: D

Solution The app was developed by Agricultural and Processed Food Products Export Development Authority for facilitating farm registration, testing, and certification of fruits and vegetables for export from India to regions such as the European Union.

The mobile app will allow farmers to apply online to facilitate their farm registration and tracking the status of application and approvals by State Government and Lab sampling by authorized Laboratories, an official statement said.

Ref: http://www.thehindu.com/business/Industry/identify•farmers•labs•via•app•technology/article19604090.ece

3. Which of the following statements is/are correct about Dr. Sarvepalli Radhakrishnan?

1. He was the First Vice President and the Second President of India. 2. He was among the first recipients of Bharat Ratna.

http://www.jigyasaias.com/Admin/printDailyTest/DTEST59AE53E02AE0F 1/5 13/09/2017 Print Question Paper

Choose the correct option.

A. 1 only

B. 2 only

C. Both 1 and 2

D. Neither 1 nor 2

Correct Answer: C

Solution Both the statements are correct.

He was the First vice president and the second President of India.

He, along with C.Rajgopalachari and C.V. Raman were the first recipient of Bharat Ratna in the year 1954.

He is known for his distinguished Career in Teaching, an accomplished scholar, a distinguished philosopher, a consummate statesman and an effective diplomat.

On his honour, His Birthday 5 September is celebrated as Teacher's Day.

Ref: http://www.iep.utm.edu/radhakri/

4. SAUNI Yojana has been launched to divert one MAFt excess flood water to Saurashtra Region.

This Project will Divert excess over flowing flood water of which River?

A. Tapti

B. Narmada

C. Mahi

D. Sabarmati

Correct Answer: B

Solution Saurashtra•Narmada Avataran Irrigation Yojana (Sauni Yojana) has been launched to divert one MAFt excess over flowing flood water of Narmada allocated to Saurashtra Region. The excess over flowing flood water of Narmada will be distributed to 115 reservoirs of eleven districts of Saurashtra through total 1126 km long four link pipelines benefitting 10,22,589 acre land.

The President of India, Shri Ram Nath Kovind laid the foundation stone of Phase•II Link•4 of this project in Rajkot, Gujarat on September 4, 2017.

Ref: PIB 4 spet Press Release.

https://guj•nwrws.gujarat.gov.in/showpage.aspx?contentid=4720&lang=english

5. Which of the following states of India does not share Boundary with Myanmar?

A.

B.

C.

D.

Correct Answer: C

Solution 4 states of India share boundary with Myanmar; i.e. Arunachal Pradesh, Nagaland, , and Mizoram. Tripura do not share boundary with Myanmar http://www.jigyasaias.com/Admin/printDailyTest/DTEST59AE53E02AE0F 2/5 13/09/2017 Print Question Paper

Ref: Map

6. Which of the following statements is/are correct about Geographical Indications (GIs):

1. A Geographical Indication or a GI is a sign used on products that have a specific geographical origin and possess qualities or a reputation that are due to that origin. 2. GI signs are given to natural growing products only not to the man made products. 3. Department of Industrial Policy and Promotion (DIPP) has recently launched a social media campaign to promote Indian Geographical Indications (GIs).

Choose the correct option.

A. 1 only

B. 1 and 3 only

C. 2 and 3 only

D. All of the above

Correct Answer: B

Solution Statement 2 is incorrect. It is awarded for man made products also.

Statement 1 and 3 are correct.

The Cell for IPR Promotions & Management (CIPAM) under the aegis of Department of Industrial Policy and Promotion (DIPP), Ministry of Commerce and Industry, has launched a social media campaign to promote Indian Geographical Indications (GIs) with #LetsTalkIP which is an ongoing movement initiated by CIPAM to make more people aware about the importance of Intellectual Property Rights.

A Geographical Indication or a GI is a sign used on products that have a specific geographical origin and possess qualities or a reputation that are due to that origin. Such a name conveys an assurance of quality and distinctiveness which is essentially attributable to its origin in that defined geographical locality. Darjeeling Tea, Mahabaleshwar Strawberry, Blue Pottery of Jaipur, Banarasi Sarees and Tirupati Laddus are some of the GIs.

Ref: PIB: CIPAM•DIPP launches social media campaign to promote Geographical Indications Press Release 04•September, 2017.

7. Advance Pricing Agreements (APAs) often remains in news. This term is related to:

A. FDI in Retail Sector

B. Direct Taxation

C. Defence Procurement

D. Public Private Partnership

Correct Answer: B

http://www.jigyasaias.com/Admin/printDailyTest/DTEST59AE53E02AE0F 3/5 13/09/2017 Print Question Paper

Solution The Advance Pricing Agreement (APA) program allows the taxpayer and the tax authority to avoid future transfer pricing disputes by entering into a prospective agreement, generally covering at least five tax years, regarding the taxpayer's transfer prices.

Recently, the Central Board of Direct Taxes (CBDT) has entered into 4 more Advance Pricing Agreements (APAs) during August 2017. Out of these 4 Agreements, 3 are Unilateral and 1 is a Bilateral. The Bilateral APA is for international transactions between an Indian company and a UK•based company. This is the 8th Bilateral APA with the United Kingdom and 13th overall (the other 5 being with Japan).

Ref: Indian Advance Pricing Agreement regime moves forward with signing of four Advance Pricing Agreements (APAs) PIB, 4 Sept 2017 Press Release.

8. Which of the following statements is correct about Wolrd Heritage Site:

1. To be included on the World Heritage List, sites must be of outstanding universal value. 2. Ahmedabad is the 3rd Indian City to be accorded the status of World Heritage city after and Madurai.

Choose the correct option:

A. 1 only

B. 2 only

C. Both 1 and 2

D. Neither 1 nor 2

Correct Answer: A

Solution The Second statement is incorrect.

There are 250 heritage cities in the world and Ahmedabad is one of them. It is the first in the country to be accorded the status.

The walled city of Ahmadabad, founded by Sultan Ahmad Shah in the 15th century, on the eastern bank of the Sabarmati river, presents a rich architectural heritage from the Sultanate period, notably the Bhadra citadel, the walls and gates of the Fort city and numerous mosques and tombs as well as important Hindu and Jain temples of later periods.

Ref: http://whc.unesco.org/en/list/1551/

9. Consider the following statements about BRICS:

1. BRICS countries come from Asia, Africa, and South America and are all members of the G20. 2. BRICS Countries accounts about 26% of world land area, 42% of world population. 3. Brazil was the last country to join BRICS,

Which of the above is/are correct?

A. 1 and 2 only

B. 2 and 3 only

C. 1 and 3 only

D. All of the above

Correct Answer: A

Solution BRICS has grown into an important platform for cooperation among emerging markets and developing countries. BRICS countries come from Asia, Africa, Europe and America and are all members of the G20. Together, they account for 26.46% of world land area, 42.58% of world population, 13.24% of World Bank voting power and 14.91% of IMF quota shares. According to IMF‘s estimates, BRICS countries generated 22.53% of the world GDP in 2015 and has contributed more than 50% of world economic growth during the last 10 years. http://www.jigyasaias.com/Admin/printDailyTest/DTEST59AE53E02AE0F 4/5 13/09/2017 Print Question Paper

Statement 3 is incorrect. South Africa was the last country to join BRICS in 2010.

Ref: https://www.brics2017.org/English/AboutBRICS/BRICS/ http://www.mea.gov.in/in•focus•article.htm?21292/BRICS+Brazil+Russia+India+China+and+South+Africa

10. Which of the following Wildlife Sanctuary and National Park is not located in Rajasthan?

A. Ranthambore National Park

B. Sariska National Park

C. Sultanpur Bird Sanctuary

D. Kumbhalgarh Wildlife Sanctuary

Correct Answer: C

Solution Sultanpur Bird Sanctuary has been declared as a National park by Haryana Government. Sultanpur Bird Sanctuary is located in Gurgaon District in Haryana State. This Bird Sanctuary, ideal for birding and bird watchers, is best visited in winters when a large number of migratory birds come here.

Ref: https://www.sultanpurbirdsanctuary.com/

http://www.jigyasaias.com/Admin/printDailyTest/DTEST59AE53E02AE0F 5/5 13/09/2017 Print Question Paper

1. In the context of internet term like, 'Locky', 'Wannna Cry', 'Peyta' etc. often remains in the news. These are:

A. Firewall Security Codes

B. Harmful Online Games

C. Ransomware

D. Online Payment Gateway

Correct Answer: C

Solution Ransomware is a type of malicious software designed to block access to a computer system until a sum of money is paid.

Locky, Peyta and Wannacry are such ransomware only.

Ref: http://indianexpress.com/article/technology/tech•news•technology/locky•everything•you•need•to•know•the•global• ransomware•attack/

2. In Articles 371•371J there are special provisions provided for various states in India.

Which of the following states does not have gets any special provisions from those articles?

1. Madhya Pradesh. 2. Gujarat. 3. Uttar Pradesh 4. Haryana

Choose the correct option.

A. 1, 3 and 4 only

B. 2, 3 and 4 only

C. 1 and 3 only

D. All of the above

Correct Answer: A

Solution Under article 371 'special responsibility' to Governor to establish 'separate development board' for Saurashtra and Kutch Region of Gujarat.

There are no special provisions for any other state mentioned above.

Ref: http://indianexpress.com/article/explained/simply•put•in•constitution•a•range•of•special•provisions•for•states•other• than•jammu•kashmir•too•4830520/ M.Laxmikanth

3. Which of the following statements is/are correct about Regional Comprehensive Economic Partnership (RCEP):

1. India is a member of this organization. 2. All the SAARC countries are the member of this Organisation. 3. All the ASEAN countries are the member of Organisation.

Choose the correct answer.

A. 1 only

B. 1 and 3 only

C. 3 only

http://www.jigyasaias.com/Admin/printDailyTest/DTEST59AFB1C4DA72B 1/4 13/09/2017 Print Question Paper

D. All of the above

Correct Answer: B

Solution The Second Statement is incorrect; not all SAARC countries are members of RCEP (in fact out of SAARC only India is a member).

RCEP is a comprehensive regional economic integration agreement amongst the 10•ASEAN countries (Brunei Darussalam, Cambodia, Indonesia, Laos, Malaysia, Myanmar, Philippines, Singapore, Thailand, and Vietnam) and its six Free Trade Agreements (FTAs) partners, viz. Australia, New Zealand, Japan, China, Korea and India.

Ref: http://asean.org/?static_post=rcep•regional•comprehensive•economic•partnership

4. Consider the following statements about Domestic•Systemically Important Bank (D•SIB):

1. SIBs are perceived as banks that are ‗Too Big To Fail'. 2. D•SIBs are mandated to maintain a progressively Common Equity Tier 1 (CET1). 3. Ministry of Finance declares a list of D•SIBs every year.

Which of the above is/are correct?

A. 1 and 2 only

B. 2 and 3 only

C. 1 and 3 only

D. All of the above

Correct Answer: A

Solution Statement 3 is incorrect the List of Domestic Systematically Important Banks are released by RBI.

Recently the Reserve Bank of India has listed HDFC Bank as a domestic systemically important bank (DSIB) under the bucketing structure identified last year. State Bank of India and ICICI Bank were identified as DSIBs under the RBI rules in 2015. The additional Common Equity Tier 1 (CET1) requirement for D•SIBs has already been phased•in from April 1, 2016

Ref: http://indianexpress.com/article/business/banking•and•finance/rbi•lists•hdfc•bank•as•systematic•important•4828903/ http://www.livemint.com/Industry/wTwW1LR4znAzt4WKjpzfZP/RBI•includes•HDFC•Bank•in•too•big•to•fail•lenders•list.html

5. Quantum Cryptography can be used for which of the following purpose:

A. Making Computer Hardware

B. Enabling Fast and Secure Communication.

C. Intergalactic Telescope.

D. Research in Dark Matter

Correct Answer: B

Solution Quantum cryptography is a new and important chapter. It is a recent technique that can be used to ensure the confidentiality of information transmitted between two parties.

Ref: https://www.economist.com/news/science•and•technology/21727889•quantum•cryptographys•early•birds•first• quantum•cryptographic•satellite

6. Which of the following statements is/are correct About BRICS 2017 Xiamen Declarations: http://www.jigyasaias.com/Admin/printDailyTest/DTEST59AFB1C4DA72B 2/4 13/09/2017 Print Question Paper

1. It had agreed to expand the BRICS and add 5 new permanent members in BRICS. 2. BRICS countries agreed to promote the development of BRICS Local Currency Bond Markets and jointly establish a BRICS Local Currency Bond Fund.

Choose the correct option.

A. 1 only

B. 2 only

C. Both 1 and 2

D. Neither 1 nor 2

Correct Answer: B

Solution The first statement is incorrect, there is no expansion of BRICS agreed upon.

The second Statement is correct.

Ref: http://pibphoto.nic.in/documents/rlink/2017/sep/p20179401.pdf

7. The Indirasagar Polavaram project is on which river?

A. Mahanadi

B. Godavari

C. Krishna

D. Penner

Correct Answer: B

Solution Indirasagar Polavaram an interstate project on river Godavari has been conceived as a part of recommendations of Godavari Water Disputes Tribunal (GWDT). GWDT finalized its award in 1980. The award identifies individual projects that can be taken up by the co•basin states of Maharashtra, Madhya Pradesh (including Chhattisgarh), Orissa, Karnataka and Andhra Pradesh (AP) on the main Godavari river as well as its tributaries.

Ref: http://wrmin.nic.in/forms/list.aspx?lid=380

8. Which of the following statements is/are correct about National Human Right Commission?

1. The salaries and allowances of Chairman and Members of NHRC are charged on the consolidated Fund of India. 2. It can only take up matters which are assigned to it by the Central Government or Supreme Court. 3. It has no power to punish the violator or award compensation to the victim.

Choose the correct option.

A. 1 and 2 only

B. 3 only

C. 1 and 3 only

D. All of the above

Correct Answer: B

Solution Statement 1 and 2 are incorrect.

The Salary and Expenditure are not charged on the Consolidated Fund, but it cannot be changed to their disadvantage. http://www.jigyasaias.com/Admin/printDailyTest/DTEST59AFB1C4DA72B 3/4 13/09/2017 Print Question Paper

And NHRC can take suo motu action as well.

statement 3 is correct.

Ref: http://www.thehindu.com/news/national/other•states/nhrc•sends•notice•to•rajasthan/article19626174.ece

M.Laxmikanth

9. Which of the following folk paintings is/are correctly matched with its State of Origin?

1. Warli Paintings: Maharashtra. 2. Madhubani Paintings: Andhra Pradesh. 3. Kalighat Paintings: West Bengal.

Choose the correct option

A. 1 and 3 only

B. 1 and 2 only

C. 3 only

D. All of the above

Correct Answer: A

Solution Warli Paintings are a folk painting style by the Warli Tribe in Maharashtra. They are different because they depict marriage ceremonies through their paintings. Stick figures, spiral patterns being formed in this painting style.

Kalighat is a famous pilgrimage of Goddess Kali in Kolkata, West Bengal. These paintings got their name from the place itself. The artists depict mythological stories through their paintings. They have their individualistic style which cannot be seen in any other painting style.

Option 2 is incorrect.

Madhubani paintings originate from Uttar Pradesh and Bihar. It is also called as Mithila Art. The stories of the artists have been evolving from the time of Janak Raja, the Mithil King who married his Daughter Sita to Lord Ram.

Ref: https://api.list.ly/list/VYY•10•folk•painting•styles•from•india

10. 'Surya Kiran' is a joint military exercise with the focus on counter•terrorism. It is an exercise between which countries?

A. India•Japan

B. India•Myanmar

C. India•USA•Japan

D. India•Nepal

Correct Answer: D

Solution The exercise • Surya Kiran is an India•Nepal joint military exercise with a focus on counterterrorism, was being participated by around 300 troops each side in Rupandehi district.

Ref: http://www.hindustantimes.com/india•news/india•nepal•joint•military•exercise•begins•with•focus•on•counter• terrorism/story•kqF8uTxahI6cEqT6DaPJHO.html

http://www.jigyasaias.com/Admin/printDailyTest/DTEST59AFB1C4DA72B 4/4 13/09/2017 Print Question Paper

1. Article 256 of the Constitution of India talks about which of the following:

1. The states shall exercise its executive power to ensure compliance with the laws made by Parliament and any existing laws which apply in that State. 2. Union shall give directions to a State as neccesary to ensure its compliance with parliamentary and state laws. 3. Power of Supreme Court to dismiss state government if it fails to comply with the order of Union Government

Choose the correct option.

A. 1 and 2 only

B. 1 and 3 only

C. 2 and 3 only

D. All of the above

Correct Answer: A

Solution Article 256 of the Constitution talks about, Obligation of States and the Union The executive power of every State shall be so exercised as to ensure compliance with the laws made by Parliament and any existing laws which apply in that State, and the executive power of the Union shall extend to the giving of such directions to a State as may appear to the Government of India to be necessary for that purpose.

Supreme Court does not have any such power as mentioned in Statement 3.

Ref: M.laxmilkanth

2. Which of the following statements is/are correct?

1. The Western and Eastern Ghats meet each other at Nilgiri Hills. 2. Gulf of Mannar marine national park is located in the Arabian sea.

Choose the correct option.

A. 1 only

B. 2 only

C. Both 1 and 2

D. Neither 1 nor 2

Correct Answer: A

Solution Gulf of Mannar marine national park is located is located in Palk Strait (between India and Srilanka) and not in the Arabian sea.

Statement 1 is correct.

Ref: Map;

http://ncert.nic.in/ncerts/l/kegy102.pdf

3. 'Rakhine' region often remains in news; it is located in which country?

A. Vietnam

B. Indonesia

C. Thailand

D. Myanmar

Correct Answer: D http://www.jigyasaias.com/Admin/printDailyTest/DTEST59B102BA1C5BB 1/5 13/09/2017 Print Question Paper

Solution Rakhine State is a state in Myanmar. Situated on the western coast of Myanmar, it is in news because of ethnic conflict in the region between Rohingya Muslims and Rakhine Buddhists.

Ref: http://indianexpress.com/article/india/rohingya•violence•all•stakeholders•must•work•to•preserve•myanmars•unity• says•pm•modi•4831436/

Map

4. The New Development Bank was formed to support infrastructure and sustainable development efforts for faster development through innovation and cutting•edge technology in the emerging economies.

It is an Initiative of Which organization?

A. ASEAN

B. East Asia Summit

C. BRICS

D. WTO

Correct Answer: C

Solution New Development Bank is an initiative of BRICS

Brief History

At the fourth BRICS Summit in New Delhi (2012), the leaders of Brazil, Russia, India, China and South Africa considered the possibility of setting up a new Development Bank to mobilize resources for infrastructure and sustainable development projects in BRICS and other emerging economies, as well as in developing countries. They directed Finance Ministers to examine the feasibility and viability of this initiative, to set up a joint working group for further study, and to report back by the next Summit in 2013.

Following the report from the Finance Ministers at the fifth BRICS summit in Durban (2013), the leaders agreed on the feasibility of establishing the New Development Bank and made the decision to do so. It was also agreed that the initial contribution to the Bank should be substantial and sufficient for it to be effective in financing infrastructure.

During the sixth BRICS Summit in Fortaleza (2014), the leaders signed the Agreement establishing the New Development Bank (NDB).

The New Development Bank was formed to support infrastructure and sustainable development efforts in BRICS and other underserved, emerging economies for faster development through innovation and cutting•edge technology. The bank will partner nations through capital and knowledge, achieving development goals with transparency and empathy and creating an equal opportunity for the development of all countries.

5. Which of the following statements is/are correct about Bahadur Shah Jafar:

1. The Mutinous Soldiers of East India Company proclaimed him as their Leader during the 1857 revolt. 2. He was sentenced to life imprisonment in the prison of Rangoon. 3. He was last Mughal ruler.

Choose the correct option.

A. 1 and 2 only

B. 3 only

C. 2 and 3 only

D. All of the above

Correct Answer: D

http://www.jigyasaias.com/Admin/printDailyTest/DTEST59B102BA1C5BB 2/5 13/09/2017 Print Question Paper

Solution All the Statements are absolutely Correct.

When the revolt of 1857 started in Meerut, the mutinous sepoy marched to Delhi and proclaimed Bahadurshah Jafar as their leader, other regional leaders of revolt also accepted his leadership, when the revolt ended he was trailed by Britishers and sentenced to imprisonment in Rangoon. He was the Last Mughal ruler, none of his descendants were accorded the status of the ruler.

Ref: http://ncert.nic.in/ncerts/l/hess105.pdf http://timesofindia.indiatimes.com/india/pm•modi•visits•mughal•ruler•bahadur•shah•zafars•grave•in• yangon/articleshow/60405523.cms

6. Which of the following tiger reserves is/are correctly matched with the state of their location?

1. Buxa Tiger Reserve: Bihar 2. Ranthambore Tiger Reserve: Rajasthan 3. Periyar Tiger Reserve: Kerala

Choose the correct option.

A. 1 and 3 only

B. 2 and 3 only

C. 2 only

D. All of the above

Correct Answer: B

Solution Buxa Tiger Reserve is located in West Bengal and not in Bihar

Others are correctly matched.

Ref: http://wiienvis.nic.in/Database/trd_8222.aspx http://www.thehindu.com/news/national/other•states/where•tigers•are•makeing•a•comeback/article19611467.ece

7. Coral Reefs in India is/are found in:

1. Lakshadweep. 2. Gulf of Kutch 3. Andaman and Nicobar

Choose the correct option.

A. 1 only

B. 3 only

C. All of the above

D. None of the above

Correct Answer: C

Solution The major reef formations in India are restricted to the Gulf of Mannar, Palk bay, Gulf of Kutch, Andaman and Nicobar Islands and the Lakshadweep islands. While the Lakshadweep reefs are atolls, the others are all fringing reefs. Patchy coral is present in the inter•tidal areas of the central west coast of the country.

Patchy coral is present in the inter•tidal areas of the central west coast of the country. Coral reefs in India are being damaged and destroyed at an increasing rate. They face serious problems of stress from anthropogenic pressures and interference.

Ref: http://www.fao.org/docrep/X5627E/x5627e06.htm

http://ncert.nic.in/ncerts/l/iess102.pdf http://www.jigyasaias.com/Admin/printDailyTest/DTEST59B102BA1C5BB 3/5 13/09/2017 Print Question Paper

8. Consider the following statements about India‘s 'zero­subsidy', Unnat Jyoti by Affordable Lighting for All (UJALA) programme:

1. India has launched this Scheme in the State of Melaka, Malaysia. 2. Under this scheme, each household will get 10 high•quality 9•watt LED bulbs at an affordable price.

Which of the above is/are correct.

A. 1 only

B. 2 only

C. Both 1 and 2

D. Neither 1 nor 2

Correct Answer: C

Solution Energy Efficiency Services Limited (EESL) , under Ministry of Power, Government of India has launched UJALA (Unnat Jyoti by Affordable Lighting for All) Scheme in the State of Melaka, Malaysia.

Under this scheme, each household in Melaka will get 10 high quality 9•watt LED bulbs at a cost of only RM 10, which is a special price and is almost half of what in begin offered in the market. The distribution of these LED bulb will take place from 28 numbers of Japerun in the region. These Japerun are a unique community welfare and engagement centers, which are situated across the Melakan State.

Ref: http://pib.nic.in/newsite/PrintRelease.aspx?relid=170569

9. Which of the following diseases is caused by 'Cadmium Pollution'?

A. Minamata

B. Itai•Itai

C. Fluorosis

D. Blue Baby Syndrome

Correct Answer: B

Solution Itai•Itai disease was caused by cadmium poisoning.

Itai•itai disease was officially recognized in 1968 as the first disease induced by environmental pollution in Japan after legal proceedings. The basis of the court decision is simply but clearly shown in "The view of the Ministry of Health and Welfare" which consisted of 7 articles and was announced on 8th May, 1968. Especially, the basis that concluded the cause of the disease to be Cd pollution was epidemiological evidence that only cadmium can explain the limited development of itai•itai disease in a specific area around parts of the Jinzu River.

Ref: http://www.kanazawa•med.ac.jp/~pubhealt/cadmium2/itaiitai•e/itai01.html http://www.dailypioneer.com/columnists/oped/heavy•metal•toxicity•and•water•contamination.html

10. The government of India has recently launched 'Diksha Portal'. This portal will be used as:

A. Portal dedicated to national heritages

B. A national digital infrastructure for Teachers.

C. Digital infrastructure to fetch Sports Talent

D. Defence infrastructure

Correct Answer: B

http://www.jigyasaias.com/Admin/printDailyTest/DTEST59B102BA1C5BB 4/5 13/09/2017 Print Question Paper

Solution On the occasion of Teachers‘ Day, Shri Venkaiah Naidu also launched DIKSHA portal, a national digital infrastructure for teachers. DIKSHA will enable, accelerate and amplify solutions in the realm of teacher education.

DIKSHA is a unique initiative which leverages existing highly scalable and flexible digital infrastructures, while keeping teachers at the center. It is built considering the whole teacher‘s life cycle ­ from the time student teachers enroll in Teacher Education Institutes (TEIs) to after they retire as teachers.

Ref: http://pib.nic.in/newsite/PrintRelease.aspx?relid=170547

http://www.jigyasaias.com/Admin/printDailyTest/DTEST59B102BA1C5BB 5/5 13/09/2017 Print Question Paper

1. Consider the following statements about Heavy Metal Poisoning:

1. Heavy metal poisoning is the accumulation of heavy metals, in toxic amounts, in the soft tissues of the body. 2. Natural causes like seepage from rocks, volcanic activity, and forest fire etc can also cause Heavy Metal Pollution. 3. Animals at the top levels of the food chain are worst affected from Heavy Metal Pollution.

Which of the above is/are correct?

A. 1 and 2 only

B. 2 and 3 only

C. 1 and 3 only

D. All of the above

Correct Answer: D

Solution Heavy metal poisoning is the accumulation of heavy metals, in toxic amounts, in the soft tissues of the body. Symptoms and physical findings associated with heavy metal poisoning vary according to the metal accumulated. Many of the heavy metals, such as zinc, copper, chromium, iron, and manganese, are essential to body function in very small amounts. But, if these metals accumulate in the body in concentrations sufficient to cause poisoning, then serious damage may occur. The heavy metals most commonly associated with poisoning of humans are lead, mercury, arsenic, and cadmium. Heavy metal poisoning may occur as a result of industrial exposure, air or water pollution, foods, medicines, improperly coated food containers, or the ingestion of lead•based paints. Natural causes like seepage from rocks, volcanic activity, and forest fire etc can also cause Heavy Metal Pollution. Heavy Metals biomagnifies up the food chain hence damaging the animal top of the food chain.

Hence, All the statements are correct.

Ref: https://rarediseases.org/rare•diseases/heavy•metal•poisoning/ http://www.dailypioneer.com/columnists/oped/heavy•metal•toxicity•and•water•contamination.html https://blog.ifis.org/2014/food•science•and•technology/heavy•metals•in•the•food•chain•could•they•be•harming•your• health

2. Which of the following statements is/are correct about Chief Information Commissioner of India?

1. It is a Constitutional Post. 2. The chief information commissioner is appointed on the recommendation of a committee consisting of Prime Minister, Leader of opposition and a Union cabinet minister.

Choose the correct option.

A. 1 only

B. 2 only

C. Both 1 and 2

D. Neither 1 nor 2

Correct Answer: B

Solution Statement 1 is incorrect, CIC is not a constitutional post, it is a statutory post under Right to Information Act, 2005.

While statement 2 is correct, As per Section 12(3) of the RTI Act 2005 the chief information commissioner is appointed by The President on the recommendation of a committee consisting of• PM•as chairman, Leader of opposition, Union cabinet ministers nominated by PM.

Ref: http://cic.gov.in/who•are•we

3. Which of the following is/are not Located in Kerala?

1. Peechi•Vazhani Wildlife Sanctuary. http://www.jigyasaias.com/Admin/printDailyTest/DTEST59B2789BADF9A 1/5 13/09/2017 Print Question Paper

2. Lawson's Bay Beech. 3. Vembanad Lake.

Choose the correct option

A. 1 only

B. 2 only

C. 3 only

D. All of the above

Correct Answer: B

Solution Peechi•Vazhani Wildlife sanctuary and Vembanad Lake are located in Kerala.

While Lawson's Bay Beach is located in Vishakhapattanam, Andhra Pradesh.

Ref: The Hindu 8 Sept Edition. http://www.forest.kerala.gov.in/index.php/wildlife/2015•03•16•09•50•24/2015•06•26•09•04•29/peechi•vazhani•wildlife• sanctuary

4. 'JAM' Trinity is derived from?

A. Just and Accounted Money

B. Jandhan Aadhar and Mobile

C. Jan Aajivika Money

D. None of the above

Correct Answer: B

Solution JAM, derives from Jan Dhan, Aadhaar and Mobile.

The JAM Trinity holds the key to one of the biggest pieces of reform ever attempted in India — direct subsidy transfers to the account of the beneficiaries.

This helps in removing middle man, misappropriation of fund, fake beneficiaries etc.

Ref: http://www.thehindu.com/opinion/lead/social•revolution•in•a•jam/article19638236.ece

5. Which of the following ranges is/are correctly matched with their state?

1. Dhaula Dhar Range: Himachal Pradesh. 2. Pir Panjal Range: Uttarakhand 3. Purvanchal Range: Uttar Pradesh.

Choose the correct option.

A. 1 only

B. 2 and 3 only

C. All of the above

D. None of the Above

Correct Answer: A

Solution Pir Panjal Range is in Jammu And Kashmir, and Purvanchal Range is in North•East. http://www.jigyasaias.com/Admin/printDailyTest/DTEST59B2789BADF9A 2/5 13/09/2017 Print Question Paper

Dhaula Dhar is correctly Matched,

Ref: http://ncert.nic.in/ncerts/l/iess102.pdf

6. Ocean Gyres are referred to:

A. Water Passage between two oceans

B. Mountain Ranges beneath the Sea

C. Volcanic Activities on the sea floor bed

D. Large Circulating System of Ocean Currents.

Correct Answer: D

Solution A Gyre is any large system of circulating ocean currents, particularly those involved with large wind movements. Gyres are caused by the Coriolis effect.

This circulation leads to accumulation of sea waste at some places which while floating looks like an Island (sometimes referred as plastic islands).

Ref: https://oceanservice.noaa.gov/education/kits/currents/05currents1.html

https://eos.org/features/monitoring•ocean•change•in•the•21st•century

7. Which of the following statements is/are correct about Contingency Fund of India?

1. It is established under the Constitution of India. 2. No money can be drawn out of the contingency fund of India without the approval of the Parliament. 3. All the Charged Expenditures are debited from this fund.

Choose the correct option?

A. 1 only

B. 2 and 3 only

C. All of the above

D. None of the Above

Correct Answer: A

Solution Statement 2 and 3 are incorrect.

Withdrawling money from the contingency fund will not require parliamentary approval. http://www.jigyasaias.com/Admin/printDailyTest/DTEST59B2789BADF9A 3/5 13/09/2017 Print Question Paper

And all the charged expenditures are debited from the Consolidated fund of India and not Contingency fund.

Statement 1 is correct, contingency fund of India is establised under the constitution under article 267(1).

Ref: M.Laxmikanth http://indianexpress.com/article/explained/size•of•contingency•fund•where•govt•rbi•disagree•4833539/

8. Recently India refused to be a part of 'Bali Declaration' in view of the fact that the declaration was not in the line of agreed principles.

This declaration is related to which organization?

A. ASEAN

B. UN Security Council

C. World Parliamentary Forum on Sustainable Development

D. Regional Comprehensive Economic Partnership

Correct Answer: C

Solution Bali Declaration was adopted at the World Parliamentary Forum on Sustainable Development.

India dissociated itself today from the declaration adopted as it carried a reference to violence in the Rakhine state of Myanmar, from where 125,000 Rohingyas have fled to Bangladesh.

An Indian parliamentary delegation, led by Speaker Lok Sabha Sumitra Mahajan, refused to be a part of the Declaration held at Nusa Dua in Indonesia.

"This was in view of the fact that the declaration, which was to be adopted at the conclusion of the Forum, was not in line with the agreed global principles of sustainable development," said a press release issued by the Lok Sabha Secretariat.

Ref: The Hindu 8 September Edition.

9. Which of the following birds are endemic to the Western Ghats?

1. Malabar Grey Hornbill 2. Crimson backed Sunbird. 3. Sandpiper

A. 1 and 3 only

B. 1 and 2 only

C. 2 and 3 only

D. All of the above

Correct Answer: B

Solution Malabar Grey Hornbill and Crimson Backed Sunbird are endemic to the Western Ghats.

The sandpipers have a cosmopolitan distribution, occurring across most of the world's land surfaces except for Antarctica and the driest deserts. A majority of the family breed at moderate to high latitudes in the Northern Hemisphere, in fact accounting for the most northerly breeding birds in the world.

Ref: The Hindu Sept 8 Edition.

10. River Cauvery originates from which state?

A. Kerala http://www.jigyasaias.com/Admin/printDailyTest/DTEST59B2789BADF9A 4/5 13/09/2017 Print Question Paper

B. Karnataka

C. Andhra Pradesh

D. Tamil Nadu

Correct Answer: B

Solution

The river Cauvery is an Inter•State river in Southern India. It is one of the major rivers of the Peninsular flowing east and running into the Bay of Bengal.

The Cauvery rises at Talakaveri on the Brahmagiri Range of Hill in the Western Ghats, presently in the Coorg district of the State of Karnataka, at an elevation of 1.341m (4,400 ft.) above mean sea level. The catchment area of entire Cauvery Basin is 81,155 sq. km. including the other basin states of Cauvery River System and their drainage areas are indicated below.

Sl.No. Name of the Basin State Catchment area in Sq.kms

1 Karnataka 34,273

2 Kerala 2,866

3 Tamil nadu 43,868

4 Karaikkal region of Pondicherry 148

Total 81,155

Ref: http://waterresources.kar.nic.in/river_systems.htm

http://www.jigyasaias.com/Admin/printDailyTest/DTEST59B2789BADF9A 5/5 13/09/2017 Print Question Paper

1. Which of the following can be used for Biometric Identification of a person?

1. Iris Scanning. 2. Finger Prints. 3. Voice Recognition.

Choose the correct answer.

A. 1 and 2 only

B. 2 and 3 only

C. 2 only

D. All of the above

Correct Answer: D

Solution Finger prints, Eyes•Ires Recognition and Voice differs from individual to individual, hence they can be used as biometrics.

Read more: The Hindu 9 Sept 2017 edition. http://www.biometricsinstitute.org/pages/types•of•biometrics.html

2. The Cocos Plate is a young oceanic tectonic plate named for Cocos Island, which rides upon it. It is located beneath?

A. Indian Ocean

B. Pacific Ocean

C. Atlantic Ocean

D. Arctic Ocean

Correct Answer: B

Solution The Cocos Plate is a young oceanic tectonic plate beneath the Pacific Ocean off the west coast of Central America, named for Cocos Island, which rides upon it. The Cocos Plate has created approximately 23 million years ago when the Farallon Plate broke into two pieces, which also created the Nazca Plate.

Ref: http://www.popsci.com/earthquake•mexico

3. Which of the following statement is/are correct about Carbon Capture and Storage?

1. It is a technology that can capture the carbon dioxide (CO2) emissions produced from the use of fossil fuels in electricity generation and industrial processes, preventing the carbon dioxide from entering the atmosphere. 2. The carbon dioxide can be stored in the geological rock formation located several kilometers below the earth's surface.

Choose the correct option.

A. 1 only http://www.jigyasaias.com/Admin/printDailyTest/DTEST59B3B532EB980 1/5 13/09/2017 Print Question Paper

B. 2 only

C. Both 1 and 2

D. Neither 1 nor 2

Correct Answer: C

Solution Both the statements are correct.

Carbon Capture and Storage (CCS) is a technology that can capture up to 90% of the carbon dioxide (CO2) emissions produced from the use of fossil fuels in electricity generation and industrial processes, preventing the carbon dioxide from entering the atmosphere.

First, capture technologies allow the separation of carbon dioxide from gases produced in electricity generation and industrial processes by one of three methods: pre•combustion capture, post•combustion capture and oxyfuel combustion.

Carbon dioxide is then transported by pipeline or by ship for safe storage. Millions of tonnes of carbon dioxide are already transported annually for commercial purposes by road tanker, ship and pipelines. The U.S. has four decades of experience of transporting carbon dioxide by pipeline for enhanced oil recovery projects.

The carbon dioxide is then stored in carefully selected geological rock formation that are typically located several kilometres below the earth's surface.

Ref: http://www.thehindu.com/news/international/india•exploring•carbon•capture•susheel•kumar/article19645151.ece

4. Aranmula Regatta is a sporting event in the state of Kerala. It is a:

A. Martial Art Event

B. Boat Race

C. Bull Race

D. Archery

Correct Answer: B

Solution The Aranmula Boat Race the oldest river boat fiesta in Kerala, the south western State of India is held during Onam (August–September). It takes place at Aranmula, near a Hindu temple dedicated to Lord Krishna and Arjuna.

Ref: http://www.thehindu.com/news/national/kerala/aranmula•regatta•held•amid•colour•and•gaiety/article19646263.ece

5. Tax Heavens generally have which of the following features:

1. Secrecy and confidentiality of financial information and the identity of the actual shareholders. 2. Strong legislative and regulative framework for book keeping and recording transactions. 3. Strong Tax Laws and high Tax to GDP ratio.

Choose the correct option.

A. 1 and 3 only

B. 2 and 3 only

C. 1 only

D. All of the above

Correct Answer: C

Solution Statement 2 and 3 are incorrect.

http://www.jigyasaias.com/Admin/printDailyTest/DTEST59B3B532EB980 2/5 13/09/2017 Print Question Paper

A tax haven is a country that offers foreign individuals and businesses a minimal tax liability in a politically and economically stable environment, with little or no financial information shared with foreign tax authorities. Tax havens do not require individuals to reside in or businesses to operate out of their countries to benefit from local tax policies.

They Generally have following features: reduced taxation • protection of information • encouraging the development of banking sector • modern media holding • the absence of currency control • the promotional advertising • favorable tax treaties.

In the majority of offshore jurisdictions, companies do not have to present annual accounts. They are not subject to the legislative or regulative framework; Hence statement 2 is incorrect. And they have no to very low tax rate hence statement 3 is also incorrect.

Ref: http://webbut.unitbv.ro/BU2014/Series%20V/BULETIN%20V/IV•02_BOTIS.pdf

6. Which of the following statements is/are correct about GST Council?

1. It is a Constitutional Body. 2. Union Finance Minister is its chairperson.

Choose the correct option.

A. 1 only

B. 2 only

C. Both 1 and 2

D. Neither 1 nor 2

Correct Answer: C

Solution Both the statements are absolutely correct,

It is a Constitutional body as per article 279A.

the GST Council which will be a joint forum of the Centre and the States, shall consist of the following members: • Union Finance Minister • Chairperson

b) The Union Minister of State, in•charge of Revenue of Finance • Member

c) The Minister In•charge of finance or taxation or any other Minister nominated by each State Government • Members

Ref: http://www.gstcouncil.gov.in/gst•council

7. Which of the following statements is/are correct about Countervailing Duties?

1. It is levied to offset subsidies provided by foreign governments. 2. It is provided for both export and import of goods and services.

Choose the correct option.

A. 1 only

B. 2 only

C. Both 1 and 2

D. Neither 1 nor 2

Correct Answer: A

Solution An import tax imposed on certain goods in order to prevent dumping or counter export subsidies.

Statement 2 is incorrect it is levied on the imports of goods only.

Ref: http://newsonair.nic.in/news.asp?cat=National&id=NN13819 http://www.jigyasaias.com/Admin/printDailyTest/DTEST59B3B532EB980 3/5 13/09/2017 Print Question Paper

8. Which of the following is/are not a Major Port of the country.

1. Kamrajar 2. Porbandar 3. Paradip 4. Port Blair

Choose the correct option.

A. 1 and 3 only

B. 2 and 4 only

C. 1, 2 and 4 only

D. All of the above

Correct Answer: B

Solution Major Ports of the countries are

West Coast

1. Kandla (Gujarat) 2. Mumbai (Maharashtra) 3. Jawaharlal Nehru (Maharashtra) 4. Marmugao (Goa) 5. New Mangalore (Karnataka) 6. Cochin (Kerala)

East Coast

1. Tuticorin (Tamil Nadu) 2. Chennai (Tamil Nadu) 3. Ennore (Tamil Nadu) 4. Visakhapatnam (Andhra Pradesh) 5. Paradip (Orissa) 6. Kolkata, Haldia (West Bengal)

As you can see, port blair and Porbandar are not among the major port.

Ref: http://pib.nic.in/newsite/PrintRelease.aspx?relid=170634 http://www.marinebuzz.com/2009/01/26/major•and•minor•ports•in•india/

9. The theme of 51st International Literacy Day announced by UNESCO is?

A. Each one teach one.

B. Literacy in a digital world

C. Literacy as everyone's Right

D. Goal 100% Literacy

Correct Answer: B

Solution The International Literacy Day is celebrated on 8th September every year throughout the world. On this day, in the year 1965 the World Congress of Ministers of Education met in Tehran for the first time to discuss the program of education at the international level.

The theme announced by UNESCO is `Literacy in a digital world‘.

Ref: http://pib.nic.in/newsite/PrintRelease.aspx?relid=170592

10. Which of the following National Park is not located in Tamil Nadu? http://www.jigyasaias.com/Admin/printDailyTest/DTEST59B3B532EB980 4/5 13/09/2017 Print Question Paper

1. Indravati National Park. 2. Mudumalai National Park 3. Guindy National Park.

Choose the correct option

A. 1 only

B. 2 and 3 only

C. 1 and 2 only

D. All of the above

Correct Answer: A

Solution Indravati national park is located in the Bijapur District of Chattisgarh and not in Tamil Nadu.

Others are located in Tamil Nadu.

Ref: http://www.indiawildliferesorts.com/national•parks/indravati•national•park.html

http://www.jigyasaias.com/Admin/printDailyTest/DTEST59B3B532EB980 5/5 13/09/2017 Print Question Paper

1. Which of the following statements is/are correct about Acharya Vinoda Bhave:

1. He was a follower of Mahatma Gandhi. 2. He was among the first persons to start Individual Satyagraha after the failure of August Offer.

Choose the correct option.

A. 1 only

B. 2 only

C. Both 1 and 2

D. Neither 1 nor 2

Correct Answer: C

Solution Both the statements are absolutely correct.

The British Colonial government had committed India into the Second World War without the consent of the Indian people. To oppose this decision by the foreign government, the Congress party decided to launch individual satyagraha.

On October 17, 1940, the Father of the Nation, Mahatma Gandhi had chosen Acharya Vinoba Bhave as the first satyagrahi (proponent of satyagraha) to start personal satyagraha (movement which meant holding to the truth) and Jawaharlal Nehru as the second.

He was a follower of Mahatma Gandhi, during the independence movement there were many leaders, some followed Gandhi, some did not and some even opposed him like B.R. Ambedkar. Hence UPSC does give some such statements asking whether the individual was a Gandhian or Not.

Ref: NCERT, Class XI.

http://ncert.nic.in/ncerts/l/hess205.pdf (http://ncert.nic.in/ncerts/l/hess205.pdf) http://ncert.nic.in/ncerts/l/jess104.pdf (http://ncert.nic.in/ncerts/l/jess104.pdf)

2. Which of the following national parks is/are located in Uttarakhand?

1. Vally of Flowers National Park. 2. Cold Desert National Park. 3. Nanda Devi National Park.

Choose the correct answer.

A. 3 only

B. 1 and 3 only

C. 1 and 2 only

D. None of the above

Correct Answer: B

Solution Cold Desert National Park is located in Himachal Pradesh and Jammu and Kashmir, Administratively, it can be said to comprise the Leh and Kargil districts of Ladakh division in J&K, Spiti region of the Lahaul and Spiti district in H.P. and a part of Kinnaur District in the state of H.P.

Valley of Flowers is a vibrant and splendid national park reposting in West . Nestled in Uttarakhand.

Nanda Devi National Park is dominated by the 7,817 m peak of Nanda Devi, India‘s second highest mountain which is approached through the Rishi Ganga gorge, one of the deepest in the world located in Uttarakhand.

Ref: http://whc.unesco.org/en/list/335 (http://whc.unesco.org/en/list/335)

http://whc.unesco.org/en/tentativelists/6055/ (http://whc.unesco.org/en/tentativelists/6055/)

3. http://www.jigyasaias.com/Admin/printDailyTest/DTEST59B650F017DDA 1/4 13/09/2017 Print Question Paper

Navika Sagar Parikrama has recently been in news. It is a:

A. Geostationary Satellite

B. Indian Regional Navigation Sattelite System

C. Solar Powered Aircraft's Circumnavigation Expedition

D. Indian Navy's all•women crew, circumnavigation expedition

Correct Answer: D

Solution Navika Sagar Parikrama is the first•ever Indian circumnavigation of the globe by an all•women crew and shall attempt to circumnavigate the globe on Indian Navy‘s sailing vessel INSV Tarini. The crew is expected to return to Goa in April 2018, on completion of the voyage. The expedition will be covered in five legs, with stop•overs at 4 ports viz. Fremantle (Australia), Lyttleton (New Zealand), Port Stanley (Falklands), and Cape Town (South Africa).

Ref: http://pib.nic.in/newsite/PrintRelease.aspx?relid=170645 (http://pib.nic.in/newsite/PrintRelease.aspx?relid=170645)

4. Bonda Tribe belongs to which region of India?

A. Mizoram

B. Andaman and Nicobar

C. Jammu and Kashmir

D. None of the Above

Correct Answer: D

Solution Bonda Tribe belongs to isolated hill regions of the Malkangiri district of southwestern Odisha, India, near the junction of the three states of Odisha, Chhattisgarh, and Andhra Pradesh.

Ref: http://www.thehindu.com/todays•paper/tp•national/data•base•soon•on•bonda•migration/article19657862.ece (http://www.thehindu.com/todays•paper/tp•national/data•base•soon•on•bonda•migration/article19657862.ece)

5. Which of the following statements is/are correct?

1. If no Party wins absolute Majority in a state Election, then Chief Minister cannot be appointed until he proves his majority on the floor of the House. 2. No•Confidence Motion can only be brought in State Legislative Assembly and not in State Legislative Council.

Choose the correct option.

A. 1 only

B. 2 only

C. Both 1 and 2

D. Neither 1 nor 2

Correct Answer: B

Solution Statement 1 is incorrect, in case of hung assembly, the governor can appoint a Chief Minister and ask him to prove his majority on the floor of the house, within a given span of time.

Statement 2 is correct, no•confidence motion can only be brought in State Legislative Assembly and not in Legislative Council. http://www.jigyasaias.com/Admin/printDailyTest/DTEST59B650F017DDA 2/4 13/09/2017 Print Question Paper

Ref: M.Laxmikanth

6. Structural Unemployment occurs due to:

A. The economy experiencing a slowdown.

B. Mismatches between the skills employers want and the skills workers have

C. Time taken by Workers Changing Jobs

D. Marginal Productivity of Labour become Zero

Correct Answer: B

Solution Structural Unemployment occurs because of an absence of demand for a certain type of worker. This typically happens when there are mismatches between the skills employers want and the skills workers have. Major advances in technology, as well as finding lower costs of labor overseas, lead to this type of unemployment. When workers lose jobs because their skills are obsolete or because their jobs are transferred to other countries, they are structurally unemployed. It's structural unemployment because the structure of the economy has changed, not because of the regular ups and downs of it.

Unemployment because of The economy experiencing a slowdown is classified as Cyclic Unemployment. Unemployment because of Time is taken by Workers Changing Jobs is Frictional Unemployment Unemployment because of Marginal Productivity of Labour becomes Zero is Disguised Unemployment. Ref: http://ncert.nic.in/ncerts/l/keec107.pdf

7. Swami Vivekanand founded which of the following organization?

A. Arya Samaj

B. Brahmo Samaj

C. Ramakrishna Mission

D. Poona Sarvajanik Sabha

Correct Answer: C

Solution Swami Vivekananda founded the Ramakrishna Mission for social service. Its ideals are based on Karma Yoga, and its governing body consists of the trustees of the Ramakrishna Math (which conducts religious work). Both Ramakrishna Math and Ramakrishna Mission have their headquarters at Belur Math.

Ref: http://www.rkmdelhi.org/about•us/ramakrishna•mission/ (http://www.rkmdelhi.org/about•us/ramakrishna•mission/)

8. Devaliya National Park is located in which state?

A. Madhya Pradesh

B. Gujarat

C. Assam

D. Kerala

Correct Answer: B

Solution Devaliya National Park is located in Gujarat.

Ref: The Hindu 11 Sept Edition http://www.jigyasaias.com/Admin/printDailyTest/DTEST59B650F017DDA 3/4 13/09/2017 Print Question Paper

9. Which of the following statements is/are correct about Participatory Notes:

1. Participatory Notes are Overseas Derivative Instruments. 2. They allow foreign investors to buy stocks listed on Indian exchanges without being registered. 3. They are regulated by Reserve Bank of India.

Choose the correct option.

A. 1 and 2 only

B. 2 and 3 only

C. All of the above

D. None of the above

Correct Answer: A

Solution P•Notes or Participatory Notes are Overseas Derivative Instruments that have Indian stocks as their underlying assets. They allow foreign investors to buy stocks listed on Indian exchanges without being registered. The instrument gained popularity as FIIs, to avoid the formalities of registering and to remain anonymous, started betting on stocks through this route. They are regulated by SEBI and not RBI.

Ref: http://economictimes.indiatimes.com/markets/stocks/policy/everything•you•want•to•know•about•p• notes/articleshow/53442336.cms (http://economictimes.indiatimes.com/markets/stocks/news/sebi•makes•p•note•norms• stricter•to•curb•black•money•in•markets/articleshow/58384722.cms)

10. Which of the following countries does not share the boundary with the Caspian Sea?

1. Iran 2. Iraq 3. Syria

Choose the correct option.

A. 1 only

B. 2 and 3 only

C. 1 and 3 only

D. All of the above

Correct Answer: B

Solution Iraq and Syria do not share the boundary with the Caspian Sea. (Iran Does)

http://www.jigyasaias.com/Admin/printDailyTest/DTEST59B650F017DDA 4/4 13/09/2017 Print Question Paper

1. Which of the following statement is/are correct about Special Status of Kashmir?

1. Financial Emergency under article 360 cannot be applied in the state of Jammu and Kashmir. 2. The provision of article 1 does not include Jammu and Kashmir. 3. Fundamental Duties are not applicable to the State of Jammu and Kashmir.

Choose the correct option.

A. 1 only

B. 2 and 3 only

C. 1 and 3 only

D. All of the above

Correct Answer: C

Solution Statement 2 is incorrect, both Constitution of India and Constitution of Jammu and Kashmir declares the state as an integral part of India hence it is included in Article 1.

Statement 1 and 3 are correct.

The Union has no power to make a proclamation of financial emergency in the State,

and Fundamental Duties are not applicable to Jammu and Kashmir. Ref: Article 370,

M.Laxmikanth

2. Which of the following pollutants are measured by the Air Quality Index?

1. Nitrogen Oxide. 2. Ozone 3. Carbon Dioxide. 4. Chlorofluorocarbon

Choose the correct option.

A. 1 and 2 only

B. 3 and 4 only

C. 1, 2 and 4 only

D. All of the above

Correct Answer: A

Solution Air Quality Index is determined on the basis of the concentration of 8 pollutants, including Particulate Matter (PM 2.5, PM 10), sulfur dioxide (SO2), nitrogen dioxide (NO2), carbon monoxide (CO), ozone (O3), ammonia (NH3) and lead (Pb).

Ref: http://aqicn.org/faq/ (http://aqicn.org/faq/)

http://pib.nic.in/newsite/PrintRelease.aspx?relid=110654 (http://pib.nic.in/newsite/PrintRelease.aspx?relid=110654)

3. Which of the following country does not share the border with Israel?

A. Jordan

B. Saudi Arabia

C. Egypt

http://www.jigyasaias.com/Admin/printDailyTest/DTEST59B7A92E9552B 1/5 13/09/2017 Print Question Paper

D. Lebanon

Correct Answer: B

Solution

Ref:

4. Which of the following statements is/are correct about Ramsar Convention?

1. It is the intergovernmental treaty that provides the framework for the conservation and wise use of wetlands and their resources. 2. India is not a party to this convention.

Choose the correct option.

A. 1 only

B. 2 only

C. Both 1 and 2

D. Neither 1 nor 2

Correct Answer: A

Solution Statement 2 is incorrect,

The convention entered into force in India on 1 February 1982.

India currently has 26 sites designated as Wetlands of International Importance (Ramsar Sites), with a surface area of 689,131 hectares.

Statement 1 is correct.

Ref: http://www.ramsar.org/about•the•ramsar•convention (http://www.ramsar.org/about•the•ramsar•convention)

5. Which of the following statements is/are correct about Right to Education?

1. It puts the obligation of the appropriate government to provide free elementary education and ensure compulsory admission and completion of elementary education to every child in the six to fourteen age group. 2. It lays down the norms and standards relating inter alia to Pupil Teacher Ratios (PTRs), buildings and infrastructure, school•working days, teacher•working hours. http://www.jigyasaias.com/Admin/printDailyTest/DTEST59B7A92E9552B 2/5 13/09/2017 Print Question Paper

3. It provides for Right of children to free and compulsory education till completion of elementary education in a neighborhood school.

Choose the correct option.

A. 1 and 2 only

B. 2 and 3 only

C. All of the above

D. None of the above

Correct Answer: C

Solution The RTE Act provides for the:

Right of children to free and compulsory education till completion of elementary education in a neighbourhood school. It clarifies that ‗compulsory education‘ means obligation of the appropriate government to provide free elementary education and ensure compulsory admission, attendance and completion of elementary education to every child in the six to fourteen age group. ‗Free‘ means that no child shall be liable to pay any kind of fee or charges or expenses which may prevent him or her from pursuing and completing elementary education. It makes provisions for a non•admitted child to be admitted to an age appropriate class. It specifies the duties and responsibilities of appropriate Governments, local authority and parents in providing free and compulsory education, and sharing of financial and other responsibilities between the Central and State Governments. It lays down the norms and standards relating inter alia to Pupil Teacher Ratios (PTRs), buildings and infrastructure, school•working days, teacher•working hours. It provides for rational deployment of teachers by ensuring that the specified pupil teacher ratio is maintained for each school, rather than just as an average for the State or District or Block, thus ensuring that there is no urban• rural imbalance in teacher postings. It also provides for prohibition of deployment of teachers for non•educational work, other than decennial census, elections to local authority, state legislatures and parliament, and disaster relief. It provides for appointment of appropriately trained teachers, i.e. teachers with the requisite entry and academic qualifications. It prohibits (a) physical punishment and mental harassment; (b) screening procedures for admission of children; (c) capitation fee; (d) private tuition by teachers and (e) running of schools without recognition, It provides for development of curriculum in consonance with the values enshrined in the Constitution, and which would ensure the all•round development of the child, building on the child‘s knowledge, potentiality and talent and making the child free of fear, trauma and anxiety through a system of child friendly and child centred learning.

Ref: http://mhrd.gov.in/rte (http://mhrd.gov.in/rte)

6. Khasi Tribe in India belongs to which region?

A. Karnataka

B.

C. Orrisa

D. Rajasthan

Correct Answer: B

Solution Garo, Khasi, Jaintia.

The Khasis inhabit the eastern part of Meghalaya, in the Khasi and Jaintia Hills. Khasis residing in Jaintia hills are now better known as Jaintias. They are also called Pnars.

The Khasis occupying the northern lowlands and foothills are generally called Bhois. Those who live in the southern tracts are termed Wars.

Again among the Wars, those living in the are called War•Khasis and those in the Jaintia Hills, War•Pnars or War•Jaintias. In the Jaintia Hills, we have Khyrwangs, Labangs, Nangphylluts, Nangtungs in the north•eastern part and in the east. In the Khasi Hills, the Lyngngams live in the north•western part. But all of them claim to have descended from the 'Ki Hynniew Trep' and are now known by the generic name of Khasi•Pnars or simply Khasis. They have the same traditions, customs, and usage with a little variation owing to geographical divisions. http://www.jigyasaias.com/Admin/printDailyTest/DTEST59B7A92E9552B 3/5 13/09/2017 Print Question Paper

Ref: http://eastkhasihills.gov.in/culture.html

7. 'Nag' Missile with Indian Armed Forces is a:

A. Inter•Continental Ballistic Missile

B. Anti•Tank Guided Missile

C. Anti•Submarine Missile

D. Multi Barrel Rocket Launcher

Correct Answer: B

Solution NAG missile is indigenously developed Anti•Tank Guided Missile.

It is one of five missile systems developed by the Defence Research and Development Organisation (DRDO) under the Integrated Guided Missile Development Program (IGMDP).

Ref: http://economictimes.indiatimes.com/news/defence/drdo•carries•out•successful•tests•of•nag• missile/articleshow/60440692.cms (http://economictimes.indiatimes.com/news/defence/drdo•carries•out•successful•tests• of•nag•missile/articleshow/60440692.cms)

8. Which of the following statements is/are correct about GST Council:

1. The Council makes recommendations on threshold limits, GST rates including the floor rates with bands, special rates etc. 2. The weightage of votes of center and states are equal in the meetings.

Choose the correct option.

A. 1 only

B. 2 only

C. Both 1 and 2

D. Neither 1 nor 2

Correct Answer: A

Solution The Second statement is incorrect Weigtage of Centre is 1/3 and 2/3 weighage is to states.

Statement 1 is correct.

Ref: http://www.gstcouncil.gov.in/gst•council (http://www.gstcouncil.gov.in/gst•council)

9. Read the following statements.

1. UNHCR is a program governed by the UN General Assembly, and the UN Economic and Social Council.

2. India is a signatory to UN refugee convention. Choose the correct option.

A. only 1

B. only 2

C. both 1 and 2

D. neither 1 nor 2

Correct Answer: A http://www.jigyasaias.com/Admin/printDailyTest/DTEST59B7A92E9552B 4/5 13/09/2017 Print Question Paper

Solution the first statement is correct.

The Office of the United Nations High Commissioner for Refugees (UNHCR), also known as the UN Refugee Agency, is a United Nations programme mandated to protect and support refugees at the request of a government or the UN itself and assists in their voluntary repatriation, local integration or resettlement to a third country. Its headquarters are in Geneva, Switzerland, and it is a member of the United Nations Development Group.

The second statement is wrong. India is not a signatory to UN refugee covention.

http://www.jigyasaias.com/Admin/printDailyTest/DTEST59B7A92E9552B 5/5 13/09/2017 Print Question Paper

1. Which Indian state shares border with both Myanmar and Bangladesh?

A. Tripura

B. Mizoram

C. Assam

D. None

Correct Answer: B

Solution Mizoram shares border with both the nations.

Bearing on tensions on the borders of Bangladesh and Myanmar security has been beefed up in Mizoram.

2. Which of the following statements is/are correct about Vishwajeet Scheme?

1. It is a scheme by Ministry of Science and Technology. 2. The scheme provides for Rs. 1,250 crore to each of the top seven IITs over a period of five years to upgrade infrastructure, hire foreign faculty and collaborate with foreign institutions.

Choose the correct option.

A. 1 only

B. 2 only

C. Both 1 and 2

D. Neither 1 nor 2

Correct Answer: B

http://www.jigyasaias.com/Admin/printDailyTest/DTEST59B8D680E54FB 1/5 13/09/2017 Print Question Paper

Solution This is a scheme by Ministry of Human Resource and Development.

The project is aimed at catapulting Indian Institutes of Technology to the top league of global academic rankings.

The scheme among other things also provides for Rs. 1,250 crore to each of the top seven IITs over a period of five years to upgrade infrastructure, hire foreign faculty and collaborate with foreign institutions.

Ref: http://www.thehindu.com/todays•paper/tp•national/vishwajeet•scheme•a•non•starter/article19673713.ece (http://www.thehindu.com/todays•paper/tp•national/vishwajeet•scheme•a•non•starter/article19673713.ece)

http://economictimes.indiatimes.com/industry/services/education/mhrds•new•project•vishwajeet•aims•to•put•iit•in•top• league•of•global•rankings/articleshow/53626151.cms (http://economictimes.indiatimes.com/industry/services/education/mhrds•new•project•vishwajeet•aims•to•put•iit•in•top• league•of•global•rankings/articleshow/53626151.cms)

3. Which of the following statements is/are correct about Bhagat Singh?

1. He was a leader of Indian National Congress's Extremist Faction. 2. Bhagat Singh and Batukeshwarr Dutt threw a harmless bomb in the Central Legislative Assembly. 3. He was sentenced in the Kakori Conspiracy Case.

Choose the correct option.

A. 1 and 2 only

B. 2 only

C. 1 and 3 only

D. All of the above

Correct Answer: B

Solution Only Statement 2 is correct.

Statement 1 is incorrect, as he was not associated with the Congress party.

Statement 3 is incorrect, as he was not involved in Kakori Conspiracy.

Ref: Class XI; Modern History old NCERT.

4. If an ordinary Bill is passed by the Lok Sabha but Rajya Sabha does not take any action on the bill for 1 year.

Which of the following possible consequences will be followed?

A. The Bill will Lapse

B. Joint Session can be called to resolve the deadlock

C. It will be assumed passed by both the houses

D. The President can still give his assent

Correct Answer: B

Solution According to the article 108. Joint sitting of both Houses in certain cases.•

(1) If after a Bill has been passed by one House and transmitted to the other House•

The Bill is rejected by the other House; or The Houses have finally disagreed as to the amendments to be made in the Bill; or More than six months elapse from the date of the reception of the Bill by the other House without the Bill being passed by it, the President may, unless the Bill has elapsed by reason of a dissolution of the House of the People, notify to the Houses by message if they are sitting or by public notification if they are not sitting, his intention to summon them to meet in a joint sitting for the purpose of deliberating and voting on the Bill: Provided that nothing in this clause shall apply to a Money Bill. http://www.jigyasaias.com/Admin/printDailyTest/DTEST59B8D680E54FB 2/5 13/09/2017 Print Question Paper

hence, in this case, Joint sitting can be called.

Ref: M.Laxmikanth

5. Which of the following is/are correct about Sunderbans:

1. It is a UNESCO natural World Heritage Site. 2. It is a Biosphere Reserve.

Choose the correct option.

A. 1 only

B. 2 only

C. Both 1 and 2

D. Neither 1 nor 2

Correct Answer: C

Solution Both the statements are correct.

The Sundarbans contain the world's largest mangrove forests and one of the most biologically productive of all natural ecosystems. Located at the mouth of the Ganges and Brahmaputra Rivers between India and Bangladesh, its forest and waterways support a wide range of' fauna including a number of species threatened with extinction. The mangrove habitat supports the single largest population of tigers in the world which have adapted to an almost amphibious life, being capable of swimming for long distances and feeding on fish, crab and water monitor lizards.

It is both a Biosphere Reserve and a Natural World Heritage Site.

Ref: http://whc.unesco.org/en/list/452 (http://whc.unesco.org/en/list/452) http://www.moef.nic.in/sites/default/files/BR%20List.pdf (http://www.moef.nic.in/sites/default/files/BR%20List.pdf)

6. 'Sir Creek is a disputed territory between?

A. Indian and Pakistan

B. India and Bangladesh

C. India, China, and Bhutan

D. Sri Lanka and Maldives

Correct Answer: A

Solution Sir Creek is a 96 km (60 mi) tidal estuary on the border of India and Pakistan. The creek, which opens up into the Arabian Sea, divides the Gujarat state of India from the Sindh province of Pakistan.

Ref: http://economictimes.indiatimes.com/news/defence/sir•creek•is•very•important•for•national•security•nirmala• sitharaman/articleshow/60464753.cms (http://economictimes.indiatimes.com/news/defence/sir•creek•is•very•important• for•national•security•nirmala•sitharaman/articleshow/60464753.cms) http://www.jigyasaias.com/Admin/printDailyTest/DTEST59B8D680E54FB 3/5 13/09/2017 Print Question Paper

7. Which of the following statements is/are correct regarding GDP calculation:

1. By income method of GDP calculation depicts the income earned by all the factors of production. 2. Net investment is added while calculating GDP on Income Method only. 3. GDP data are released by Central Statistics Office (CSO).

Choose the correct option.

A. 1 and 2 only

B. 2 and 3 only

C. 1 and 3 only

D. All of the above

Correct Answer: C

Solution GDP can be calculated by 3 methods i.e. Income Method, Expenditure method and Production method.

Under Income Method: GDP= total wages salaries of workers + total profit to entrepreneurs + total Rent to land and Total Intrest to the Capital. Under Expenditure Method GDP = Total consumption + Investment + Government Consumption expenditure + Net exports Under Production method.

GDP= Sum of all goods produced.

as you can see investment is added while calculating GDP on Expenditure method.

Ref: Class XII Economics NCERT http://ncert.nic.in/ncerts/l/leec102.pdf (http://ncert.nic.in/ncerts/l/leec102.pdf)

8. Which of the following is/are a direct Tax?

1. Capital Gain Tax. 2. Minimum Alternative Tax. 3. Corporate Income Tax.

Choose the correct option.

A. 1 and 2 only

B. 2 and 3 only

C. 1 and 3 only

D. All of the above

Correct Answer: D

Solution Direct Taxes are those which have to be paid by the person on whom it is levied and its burden cannot be passed on, this includes Income tax, Capital Gain Tax, Corporate Tax, etc.

Ref: http://mospi.nic.in/sites/default/files/Statistical_year_book_india_chapters/DIRECT•INDIRECT%20TAX•WRITEUP.pdf (http://mospi.nic.in/sites/default/files/Statistical_year_book_india_chapters/DIRECT•INDIRECT%20TAX•WRITEUP.pdf)

9. Which of the following statements is/are correct?

1. Bye•Elections for Parliamentary Seats are conducted by the Election Commission of India. 2. Bye•Election for State Legislative Assembly seats are conducted by the State Election Commission. 3. The option of NOTA and VVPAT system cannot be provided for Bye•Election. http://www.jigyasaias.com/Admin/printDailyTest/DTEST59B8D680E54FB 4/5 13/09/2017 Print Question Paper

Choose the correct option.

A. 1 only

B. 2 and 3 only

C. All of the above

D. None of the above

Correct Answer: A

Solution Statement 2 and 3 are incorrect,

Bye•Election for State Legislative Assembly is also conducted by Election Commission of India only Statement 3 is incorrect as the process remains same as normal Election.

Ref: http://pib.nic.in/newsite/PrintRelease.aspx?relid=170727 (http://pib.nic.in/newsite/PrintRelease.aspx?relid=170727)

10. Which of the following country is not a permanent member of UN Security Council?

A. France

B. Britain

C. Germany

D. Russia

Correct Answer: C

Solution 5 permanent ent members of UN security Council are ; USA, Russia, France The United Kingdom and China.

Ref: http://www.un.org/en/sc/members/ http://www.thehindu.com/news/international/un•approves•watered•down•new•sanctions•against•north• korea/article19669985.ece (http://www.thehindu.com/news/international/un•approves•watered•down•new•sanctions• against•north•korea/article19669985.ece)

http://www.jigyasaias.com/Admin/printDailyTest/DTEST59B8D680E54FB 5/5 9/25/2017 Print Question Paper

1. The preamble of Indian Constitution mentions equality of:

1. Status 2. Expression 3. Opportunity 4. Belief

Choose the correct option.

A. 3 only

B. 2 and 4 only

C. 1 and 3 only

D. All of the above

Correct Answer: C

Solution Preamble of India provides for Equality of status and of opportunity; Full text or Preamble is:

WE, THE PEOPLE OF INDIA, having solemnly resolved to constitute India into a SOVEREIGN SOCIALIST SECULAR DEMOCRATIC REPUBLIC and to secure to all its citizens: JUSTICE, social, economic and political;

LIBERTY of thought, expression, belief, faith, and worship;

EQUALITY of status and of opportunity;

and to promote among them all

FRATERNITY assuring the dignity of the individual and the unity and integrity of the Nation;

IN OUR CONSTITUENT ASSEMBLY this twenty-sixth day of November 1949, do HEREBY ADOPT, ENACT AND GIVE TO OURSELVES THIS CONSTITUTION.

2. Which of the following good and services are outside the Purview of GST?

1. Alcohol for Human Consumption. 2. Petroleum Products 3. Mobile Communication.

Choose the correct option.

A. 1 only

B. 1 and 2 only

C. 2 and 3 only

D. All of the above

Correct Answer: B

Solution Mobile communication is taxable under GST, (@5%) While Alcohol for Human Consumption is outside its purview. http://www.jigyasaias.com/Admin/printDailyTest/DTEST59BA4637B5FCF 1/5 9/25/2017 Print Question Paper

Ref: http://www.deccanchronicle.com/business/economy/130917/daily-petrol-price-change-to-remain-gst-only-way-to- rational-pricing-pradhan.html (http://www.deccanchronicle.com/business/economy/130917/daily-petrol-price-change-to- remain-gst-only-way-to-rational-pricing-pradhan.html) http://economictimes.indiatimes.com/what-is-goods-and-service-tax-gst-and-which-are-the-commodities-proposed-to-be- kept-outside-the-purview-of-gst/articleshow/58590691.cms (http://economictimes.indiatimes.com/what-is-goods-and- service-tax-gst-and-which-are-the-commodities-proposed-to-be-kept-outside-the-purview-of-gst/articleshow/58590691.cms)

3. Which of the following statements is/are correct about Chakma and Hajong:

1. They are refugees which came to India from Jaffna region of Sri Lanka fleeing religious Prosecution. 2. They have settled in the North-East States of India.

Choose the correct option.

A. 1 only

B. 2 only

C. Both 1 and 2

D. Neither 1 nor 2

Correct Answer: B

Solution Statement 1 is incorrect.

The Chakmas and Hajongs, originally residents of the Chittagong Hill Tracts of the former East Pakistan, had to flee when their land was submerged by the Kaptai dam project in the 1960s. Buddhists by faith, the Chakmas faced religious persecution in East Pakistan along with the Hajongs, who are Hindus.

Out of those who reached India, most of them were Chakmas and only 2,000 were Hajong. The groups entered India through what was then the Lushai Hills district of Assam (today‘s Mizoram). While some stayed back with Chakmas already living in the Lushai Hills, the Indian government moved a majority of the refugees to present-day Arunachal Pradesh.

Ref: http://www.thehindu.com/news/national/who-are-chakmas/article19682129.ece http://indianexpress.com/article/who-is/who-are-chakma-and-hajong-refugees-in-arunachal-pradesh- 4841615/ (http://www.thehindu.com/news/national/who-are-chakmas/article19682129.ece)

4. Which of the following statements is/are correct about None of the Above (NOTA) option in Election.

1. The option of NOTA is only available to the electors during Lok Sabha Election. 2. If most of the votes in a general election goes to NOTA then none of the candidates stand elected and elections are re-conducted with different sets of candidates.

Choose the correct option.

A. 1 only

B. 2 only

C. Both 1 and 2

D. Neither 1 nor 2

Correct Answer: D

Solution Both the statements are incorrect,

NOTA option is also available for the election of Rajya Sabha, State Legislative assembly etc. http://www.jigyasaias.com/Admin/printDailyTest/DTEST59BA4637B5FCF 2/5 9/25/2017 Print Question Paper

and, even if NOTA gets the majority of votes, it will not affect election as the candidates with the second highest votes selected elected.

Ref: http://www.thehindu.com/todays-paper/tp-national/plea-against-nota-not-valid-says-ec/article19679940.ece (http://www.thehindu.com/todays-paper/tp-national/plea-against-nota-not-valid-says-ec/article19679940.ece)

5. Pirotan Island is a Marine National Park located in which state?

A. Gujarat

B. Kerala

C. Tamil Nadu

D. Odisha

Correct Answer: A

Solution Pirotan Island is an Arabian Sea island in the Marine National Park, Jamnagar District of Gujarat state, India. It is located 12 nautical miles (22 km), off the coast consists of mangroves and low-tide beaches,[3] and has an area of 3 square kilometers. Rozi island is located about 10 kilometers to the southeast.

Ref: http://www.thehindu.com/news/national/other-states/on-this-island-its-the-dew-of-the-jackal/article19653327.ece (http://www.thehindu.com/news/national/other-states/on-this-island-its-the-dew-of-the-jackal/article19653327.ece)

6. Which of the following statements is/are correct about Atal Mission for Rejuvenation and Urban Transformation (AMRUT) Scheme:

1. This scheme aims at Providing basic services to households and build amenities in cities. 2. It will cover all Cities and Towns with a population of over one lakh with notified Municipalities.

Choose the correct option.

A. 1 only

B. 2 only

C. Both 1 and 2

D. Neither 1 nor 2

Correct Answer: C

Solution Both the statements are absolutely correct.

Providing basic services (e.g. water supply, sewerage, urban transport) to households and build amenities in cities which will improve the quality of life for all, especially the poor and the disadvantaged is a national priority Five hundred cities will be taken up under AMRUT. The list of cities will be notified at the appropriate time. The category of cities that will be covered in the AMRUT is given below: i. All Cities and Towns with a population of over one lakh with notified Municipalities, including Cantonment Boards (Civilian areas), ii. All Capital Cities/Towns of States/ UTs, not covered in above, iii. All Cities/ Towns classified as Heritage Cities by MoUD under the HRIDAY Scheme, iv. Thirteen Cities and Towns on the stem of the main rivers with a population above 75,000 and less than 1 lakh, v. Ten Cities from hill states, islands and tourist destinations (not more than one from each State).

Ref: http://amrut.gov.in/writereaddata/The%20Mission.pdf (http://amrut.gov.in/writereaddata/The%20Mission.pdf) http://www.jigyasaias.com/Admin/printDailyTest/DTEST59BA4637B5FCF 3/5 9/25/2017 Print Question Paper

7. Which of the following statements is/are not correct about Mughal Rulers:

1. The arrival of Babur led to the establishment of Timurid Dynasty in India. 2. Fatehpur Sikri was a city founded by Shahjahan. 3. Aurangzeb was the last Crowned Mughal Ruler.

Choose the correct option.

A. 1 only

B. 2 and 3 only

C. 1 and 2 only

D. All of the above

Correct Answer: B

Solution Statement 2 and 3 are incorrect.

Fatehpur Sikri was a city established by Akbar, (and not Shahjahan).

And there were many Mughal Rulers to be crowned after Aurangzeb, though we can say that Aurangzeb was last effective Mughal Ruler, but there is a long series of Later Mughal, who ruled a weak and declining empire, ending with Bahadurshah Zafar.

Only Statement 1 is correct, as Babur belonged to Timur Dynasty.

Ref: Class XI History NCERT

8. Which of the following statements is/are correct about National Commission for Backward Caste (NCBC)

1. The commission considers inclusions in and exclusions from the lists of communities notified as backward, Schedule caste and Schedule Tribe for the purpose of job reservations and Admission in Educational Institutes. 2. Article 340 of the Constitution provides for the appointment of a Commission to investigate the conditions of and the difficulties faced by the socially and educationally backward classes and to make appropriate recommendations.

Choose the correct option.

A. 1 only

B. 2 only

C. Both 1 and 2

D. Neither 1 nor 2

Correct Answer: B

Solution Statement 1 is incorrect, The Commission only looks for the matter of Backward Caste and not for the Schedule Caste and Schedule Tribe.

Statement 2 is correct. Article 340 says, The President may by order appoint a Commission consisting of such persons as he thinks fit to investigate the conditions of socially and educationally backward classes within the territory of India and the difficulties under which they labour and to make recommendations as to the steps that should be taken by the Union or any State to remove such difficulties and to improve their condition and as to the grants that should be made for the purpose by the Union or any State the conditions subject to which such grants should be made, and the order appointing such Commission shall define the procedure to be followed by the Commission http://www.jigyasaias.com/Admin/printDailyTest/DTEST59BA4637B5FCF 4/5 9/25/2017 Print Question Paper

Ref: http://www.ncbc.nic.in/User_Panel/UserView.aspx?TypeID=1114 (http://www.ncbc.nic.in/User_Panel/UserView.aspx? TypeID=1114)

9. Amnesty International is a

A. Human Rights agency of UNO

B. Organisation under the European Union

C. World Bank Funded Refugee Agency

D. A global Human Rights

Movement Correct Answer: D

Solution Amnesty International is a global movement it is an NGO, independent of any government or International organisation's control.

Ref: https://www.amnesty.org/en/who-we-are/

10. Pradhan Mantri Jan Dhan Yojana is primarily aimed to:

A. Digital Currency

B. Financial Inclusion

C. Targeted Public Distribution

D. Poverty Alleviation

Correct Answer: B

Solution Though in many ways PMJDY is aimed to achieve all of the above-mentioned objectives.

But its primary aim is to provide Financial Inclusion by opening bank accounts for those who were earlier out of the banking system. Other objectives only come after it.

Ref: https://www.pmjdy.gov.in/about

http://www.jigyasaias.com/Admin/printDailyTest/DTEST59BA4637B5FCF 5/5 9/25/2017 Print Question Paper

1. 'Operation Indaniyat' has recently been in news. It is related to:

A. Confidence Building Measures of Indian Army in Kashmir Valley

B. Humanitarian Assistance to Bangladesh on Account of Influx of Refugees

C. Character Building to be taught as a Subject under CBSE curriculum

D. India-Pakistan tier 2 diplomatic efforts to improve ties.

Correct Answer: B

Solution Operation Insaaniyat is a Humanitarian Assistance to Bangladesh on Account of Influx of Refugees

A humanitarian crisis arose in Bangladesh due to heavy influx of refugees from neighbouring Myanmar. The government of India has decided to assist Bangladesh in this crisis by sending relief material. The Indian Air Force was tasked to airlift the relief material from India to Bangladesh. One C-17 Globemaster strategic heavy lift cargo aircraft was positioned at short notice at Delhi on 13 Sep 17 to airlift 55 Tons of relief material to Chittagong, Bangladesh. This aircraft was loaded overnight with relief material consisting of critical daily necessities viz., rice, pulses, sugar, salt, cooking oil, ready to eat meals, mosquito nets etc.

Ref: http:// http://mea.gov.in/press-releases.htm? dtl/28944/Operation_Insaniyat__Humanitarian_assistance_to_Bangladesh_on_account_of_influx_of_refugees (http:// http://mea.gov.in/press-releases.htm? dtl/28944/Operation_Insaniyat__Humanitarian_assistance_to_Bangladesh_on_account_of_influx_of_refugees)

2. Which of the following statements is/are correct about National Institutional Ranking Framework (NIRF) launched by the Ministry of Human Resource Development?

1. This framework outlines a methodology to rank Education institutions across the country. 2. Its parameters broadly cover ―Teaching, Learning and Resources,‖ ―Research and Professional Practices,‖ ―Graduation Outcomes,‖ ―Outreach and Inclusivity,‖ and ―Perception‖.

Choose the correct option.

A. 1 only

B. 2 only

C. Both 1 and 2

D. Neither 1 nor 2

Correct Answer: C

Solution Both the statements are absolutely correct.

The National Institutional Ranking Framework (NIRF) was approved by the MHRD and launched by Honourable Minister of Human Resource Development on 29th September 2015.

This framework outlines a methodology to rank institutions across the country. The methodology draws from the overall recommendations broad understanding arrived at by a Core Committee set up by MHRD, to identify the broad parameters for ranking various universities and institutions. The parameters broadly cover ―Teaching, Learning and Resources,‖ ―Research and Professional Practices,‖ ―Graduation Outcomes,‖ ―Outreach and Inclusivity,‖ and ―Perception‖.

Each broad head has an overall weight assigned to it. Within each head, the various sub-heads also have an appropriate weight distribution.

Ref: https://www.nirfindia.org/Docs/Ranking_Methodology_And_Metrics_2017.pdf (https://www.nirfindia.org/Docs/Ranking_Methodology_And_Metrics_2017.pdf) http://www.jigyasaias.com/Admin/printDailyTest/DTEST59BBA82DE59F7 1/5 9/25/2017 Print Question Paper

3. Which of the following statements is/are correct about the Global Human Capital Index?

1. The list compiled by Geneva-based World Economic Forum. 2. It ranks countries on how well they are developing their human capital on a scale across four thematic dimensions— capacity, deployment, development and know-how. 3. India ranks top in Global Human Capital Index among BRICS countries.

Choose the correct option.

A. 1 and 2 only

B. 2 and 3 only

C. 1 and 3 only

D. All of the above

Correct Answer: A

Solution Statement 3 is incorrect, India ranks 103 in the index which is far lower than its BRICS peers, with Russian Federation placed as high as 16th place, followed by China at 34th, Brazil at 77th and South Africa at 87th place.

statement 1 and 2 are absolutely correct.

Ref: https://weforum.ent.box.com/s/dari4dktg4jt2g9xo2o5pksjpatvawdb (https://weforum.ent.box.com/s/dari4dktg4jt2g9xo2o5pksjpatvawdb)

4. In which of the following formal/informal Organisations both India and Japan are members:

1. G4 Nations 2. Regional Comprehensive Economic Corporation. 3. Trans Pacific Partnership Agreement. 4. East Asian Summit.

Choose the correct option.

A. 1 and 3 only

B. 2 and 4 only

C. 1, 2 and 4 only

D. All of the above

Correct Answer: C

Solution India and Japan both are members in G4, RCEP, and EAS.

G4 is an informal grouping of 4 powerful nation i.e. India, Brazil, Germany, and Japan which wants the expansion of UN security council and support each other's bid for permanent seat in the council.

RCEP members include ASEAN countries + India, China, Japan, Australia, New Zealand and South Korea.

and. EAS members are ASEAN + Australia, China, India, Japan, New Zealand, Republic of Korea, Russian Federation and the USA.

while India is not a member of TPP.

Ref: http://www.mea.gov.in/aseanindia/about-eas.htm (http://www.mea.gov.in/aseanindia/about-eas.htm) http://www.jigyasaias.com/Admin/printDailyTest/DTEST59BBA82DE59F7 2/5 9/25/2017 Print Question Paper

5. Kole Wetlands is one of largest, highly productive and threatened wetlands, and has been declared a Ramsar Convention site for protection.

It is located in which state

A. Gujarat

B. Kerala

C. Maharashtra

D. Andaman and Nicobar

Correct Answer: B

Solution The word Kole is a Malayalam word, and means that a bumper yield. It is a particular cultivation method adopted in wastelands from December to May which otherwise is submerged from June to November, half of the year. The Kole wetlands are low lying tracts located 0.5 to 1m below Mean Sea Level (MSL) and remain submerges for about six months in a year. The average annual rainfall is 3,200 mm and temperature varied from 28°C to 31°5°C.The Kole Wetlands cover an area of about 13,632 hectares spread over Thrissur district and Malappuram district. The area extends from Chalakudy River in South to Bharathappuzha River in the North, and to Ponnani Taluk. The Kole Wetlands acts as natural drainage system for Thrissur city and Thrissur district through a network of canals and ponds which connects different parts of Kole Wastelands to river and then to the Arabian sea. It is fertile with Alluvium soil which is deposited Kechery and Karuvannoor river in the monsoon.

Ref: https://www.arcgis.com/home/item.html?id=7da1ee00ba13433aba2bc796effa9863 (https://www.arcgis.com/home/item.html?id=7da1ee00ba13433aba2bc796effa9863)

6. '2thinknow' is a research firm that specializes in analyzing innovative cities.

It recently ranked 85 cities keeping in 10 factors in mind such as innovative data sets, start-ups, patents

etc. Which of the following statements is/are correct about its list of top 25 High-Tech cities.

1. Delhi is India's most High Tech City. 2. Bangluru is named among top 25 high-tech cities of the world climbed rapidly from its ranking last year.

Choose the correct answer.

A. 1 only

B. 2 only

C. Both 1 and 2

D. Neither 1 nor 2

Correct Answer: B

Solution Statement 1 is incorrect Bangluru at the 19th spot is the only Indian city to feature in the top 25 list, hence it is the most high-tech city of India and not Delhi.

Research firm 2thinknow that specializes in analyzing innovative cities has placed Bengaluru at 19th place ahead of Berlin, Hong Kong and Shenzhen. San Francisco is at the top. The Ministers said the global recognition for Bengaluru -- the only Indian city to figure in the list -- was a vindication of the government‘s initiatives.

Ref: http://economictimes.indiatimes.com/news/economy/indicators/bengalurus-ranking-among-25-best-hi-tech-cities-in- the-world/articleshow/60482074.cms (http://economictimes.indiatimes.com/news/economy/indicators/bengalurus-ranking- among-25-best-hi-tech-cities-in-the-world/articleshow/60482074.cms) http://www.jigyasaias.com/Admin/printDailyTest/DTEST59BBA82DE59F7 3/5 9/25/2017 Print Question Paper

7. Which of the following statements is/are correct about Bitcoins?

1. Bitcoin is a consensus network that enables a new payment system and a completely digital money. 2. Bitcoin uses peer-to-peer technology to operate with no central authority or banks. 3. It is illegal in India to sale and purchase Bitcoins.

Choose the correct option

A. 1 only

B. 2 and 3 only

C. 1 and 2 only

D. 1 and 3 only

Correct Answer: C

Solution Statement 3 is incorrect,

It is not illegal to trade in Bitcoins in India, but it is not regulated hence RBI is concerned about it.

Statement 1 and 2 are absolutely correct. Bitcoin is a consensus network that enables a new payment system and a completely digital money. It is the first decentralized peer-to-peer payment network that is powered by its users with no central authority or middlemen.

Ref: http://www.thehindu.com/business/Industry/rbi-is-not-comfortable-with-bitcoins-says- sen/article19677964.ece https://bitcoin.org/en/faq (http://www.thehindu.com/business/Industry/rbi-is-not- comfortable-with-bitcoins-says-sen/article19677964.ece)

8. Which of the following statements is/are correct about GST?

1. All services provided by the government or local authority are exempt from payment of GST. 2. Government and Local authorities are liable to pay GST on services they receive.

Choose the correct option.

A. 1 only

B. 2 only

C. Both 1 and 2

D. Neither 1 nor 2

Correct Answer: B

Solution Statement 1 is incorrect, all services provided by the government are not exempt from GST, such as services of the department of Post, agency services by any department, transport of goods, services related to aircraft etc. are liable to tax.

Statement 2 is correct, Government is liable to pay taxes on services they receive until exempted under GST.

Ref: GST FAQ released on 15 Sept 2017.

9. A. Kuiper Belt

http://www.jigyasaias.com/Admin/printDailyTest/DTEST59BBA82DE59F7 4/5 9/25/2017 Print Question Paper

NASA's Cassini spacecraft is one of the most successful missions lasted for 2 decades.

This mission was aimed at exploring:

B. Jupiter and its Moons

C. Mars's atmosphere and its surface

D. Saturn and its moons

Correct Answer: D

Solution The Cassini mission to Saturn is one of the most ambitious efforts in planetary space exploration ever mounted. A joint endeavor of NASA, the European Space Agency (ESA) and the Italian space agency, Agenzia Spaziale Italiana (ASI), Cassini is a sophisticated robotic spacecraft orbiting the ringed planet and studying the Saturnian system in detail. Cassini also carried a probe called Huygens, which parachuted to the surface of Saturn‘s largest moon, Titan, in January 2005 and returned spectacular results.

Ref: https://www.nasa.gov/mission_pages/cassini/whycassini/index.html (https://www.nasa.gov/mission_pages/cassini/whycassini/index.html)

10. Which of the following statements is/are correct about State Human Rights Commission:

1. Every state is constitutionally bound to set up a Human Right Commission. 2. State Human Rights commission has the power to punish the perpetrators of Human Rights Violation and grant compensation to the victims.

Choose the correct option.

A. 1 only

B. 2 only

C. Both 1 and 2

D. Neither 1 nor 2

Correct Answer: D

Solution Both the statements are incorrect, the State Human Rights commission is not a constitutional body and the states are not constitutionally bound to set up a State Human Rights Commission.

State Human Rights commission's role is only recommendatory in nature hence it does not have the power to punish or grant compensation, it can only recommend state government to do so.

Ref: http://newsonair.nic.in/news.asp?cat=State&id=ST9996 (http://newsonair.nic.in/news.asp?cat=State&id=ST9996)

http://www.jigyasaias.com/Admin/printDailyTest/DTEST59BBA82DE59F7 5/5 9/25/2017 Print Question Paper

1. Which of the following statements is/are correct about Foreign Exchange Reserve of the country?

1. Forex reserves include foreign currency assets held by the central banks of countries. 2. It can also be held in the form of Gold, Marketable Securities, special drawing rights (SDRs) and reserve position in the IMF, etc. by the RBI 3. India holds the largest foreign Exchange reserve among BRICS nation.

Choose the correct option.

A. 1 and 2 only

B. 1 and 3 only

C. 2 only

D. All of the above

Correct Answer: A

Solution Statement 1 and 2 are absolutely correct. Forex reserves are foreign currency assets held by the central banks of countries. These assets include foreign marketable securities, monetary gold, special drawing rights (SDRs) and reserve position in the IMF. The main purpose of holding foreign exchange reserves is to make international payments and hedge against exchange rate risks.

Ref: https://www.rbi.org.in/scripts/WSSView.aspx?Id=21654 (https://www.rbi.org.in/scripts/WSSView.aspx?Id=21654)

Statement 3 is incorrect. India just reached 400 bn Foreign Reserve which is 8th Highest in the world, But among BRICS nations Russia (424 bn) and China (3000 bn) Holds more reserve than India.

Ref: http://economictimes.indiatimes.com/markets/forex/indias-forex-reserves-top-400-billion-for-first-time- ever/articleshow/60531270.cms (http://economictimes.indiatimes.com/markets/forex/indias-forex-reserves-top-400- billion-for-first-time-ever/articleshow/60531270.cms)

2. Which of the following statements is/are correct about Registration of Political Parties in India?

1. For an association or body of individual calling itself a political party and intending to avail itself of the provisions of the Representation of the People Act is required to get itself registered with the Election Commission of India. 2. Only a party recognized by the Election Commission as a state party can contest state election.

Choose the correct option.

A. 1 only

B. 2 only

C. Both 1 and 2

D. Neither 1 nor 2

Correct Answer: A

Solution Statement 1 is correct. An association or body of individual citizens of India calling itself a political party and intending to avail itself of the provisions of Part-IV-A of the Representation of the People Act, 1951, (relating to registration of political parties) is required to get itself registered with the Election Commission of India.

http://www.jigyasaias.com/Admin/printDailyTest/DTEST59BCFFC09E54E 1/6 9/25/2017 Print Question Paper

The candidates set up by a political party registered with the Election Commission of India will get preference in the matter of allotment of free symbols vis-à-vis purely independent candidates. Further, registered political parties, in course of time, can get recognition as `State Party‘ or National Party‘ subject to the fulfillment of the conditions prescribed by the Commission in the Election Symbols (Reservation and Allotment) Order, 1968, as amended from time to time.

But, statement 2 is incorrect, even a party not recognized by the election commission as state party can contest the election for state legislative assembly.

Ref: http://eci.nic.in/eci_main1/RegisterationPoliticalParties.aspx (http://eci.nic.in/eci_main1/RegisterationPoliticalParties.aspx)

3. Which of the following statements is/are correct about the Monsoon?

1. During the Monsoon months, Cyclones in the Western Pacific move westwards towards India and aid rain- bearing system over sub-continent. 2. El Nino years are those when sea surface temperatures in the eastern equatorial Pacific Rise, and it often dampens the Monsoon.

Choose the correct option

A. 1 only

B. 2 only

C. Both 1 and 2

D. Neither 1 nor 2

Correct Answer: C

Solution Both the statements are absolutely correct.

During the monsoon months, cyclones in the Western Pacific move westwards towards India and aid rain-bearing systems over the sub-continent. But during some years they ‗recurve‘, or start to swing north-east, and do not give as much of a push to the rains as they do in the good monsoon years.

This re-curving frequently happens during the El Nino years but this time it inexplicably occurred when an El Nino hasn‘t yet taken shape. El Nino years are those when sea surface temperatures in the east equatorial Pacific rise, and often dampen the monsoon. While several international meteorological agencies had raised concerns about a likely El Nino forming in August or September this year, it didn‘t happen.

Ref: http://www.thehindu.com/news/national/recurving-cyclones-dried-august-rains/article19694070.ece (http://www.thehindu.com/news/national/recurving-cyclones-dried-august-rains/article19694070.ece)

4. Current Account Deficit of a country is:

A. Difference between Import and Export of Goods only

B. Difference between imports and exports of both goods and services

C. Difference between Import and export of Services only

D. Net increase / Decrease of Forex Reserve of the Country.

Correct Answer: B

Solution Current account deficit is a measurement of a country‘s trade where the value of the goods and services it imports exceeds the value of the goods and services it exports. The current account also includes net income, such as interest and dividends, as well as transfers, such as foreign aid, though these components make up only a small percentage of the http://www.jigyasaias.com/Admin/printDailyTest/DTEST59BCFFC09E54E 2/6 9/25/2017 Print Question Paper

current account when compared to exports and imports. The current account is essentially a calculation of a country‘s foreign transactions and, along with the capital account, is a component of a country‘s balance of payment.

Though it has bearing on the Forex Reserve of the country, but it is not the only factor responsible for it.

Ref: http://www.livemint.com/Money/UL0weDmw7Bb1vEIEuR7IqM/Current-account-deficit-widens-to-24-of-GDP-in-Q1- shows-R.html (http://www.livemint.com/Money/UL0weDmw7Bb1vEIEuR7IqM/Current-account-deficit-widens-to-24-of- GDP-in-Q1-shows-R.html)

5. Which of the following statements is/are incorrect?

1. Rohingyas are a minority Buddhist community in the Rakhine State of Myanmar. 2. India shares its largest land border with Bangladesh. 3. Assam is the only state in India to border both Bangladesh and Myanmar.

Choose the right answer.

A. 1 and 2 only

B. 1 and 3 only

C. 2 only

D. All of the above

Correct Answer: B

Solution Statement 1 and 3 are incorrect

Rohingyas are a minority Muslim Community.

Assam does not share a boundary with Myanmar. It is Mizoram which shares the boundary with both Bangladesh and Myanmar.

Statement 2 is correct, India Share a boundary with 4096 KM.

Ref: http://www.thehindu.com/todays-paper/bjp-states-tell-police-to-push-back-rohingya/article19695235.ece (http://www.thehindu.com/todays-paper/bjp-states-tell-police-to-push-back-rohingya/article19695235.ece)

6. Which of the following statements is/are correct?

1. Salary and Allowances of Members of State Legislative Assemblies are determined by the Parliament. 2. MLA's of All states gets equal salaries.

Choose the correct answer.

A. 1 only

B. 2 only

C. Both 1 and 2

D. Neither 1 nor 2

Correct Answer: D

Solution Both the statements are incorrect.

Salaries of Member of State Legislative Assemblies are decided by Legislative Assembly only. http://www.jigyasaias.com/Admin/printDailyTest/DTEST59BCFFC09E54E 3/6 9/25/2017 Print Question Paper

hence they vary from one state to another.

Ref: http://timesofindia.indiatimes.com/city/bhubaneswar/odisha-cabinet-approves-salary-hike-of-mlas- ministers/articleshow/60501252.cms (http://timesofindia.indiatimes.com/city/bhubaneswar/odisha-cabinet-approves- salary-hike-of-mlas-ministers/articleshow/60501252.cms)

M,Laxmikanth

7. Which of the following statements is/are correct regarding India's trade and commerce.

1. The external debt includes NRI deposits. 2. International Debt Statistics is an inter-country comparison of debt, published by IMF. 3. India is a net Exporter of Services.

Choose the right answer.

A. 1 and 2 only

B. 2 and 3 only

C. 1 and 3 only

D. All of the above

Correct Answer: C

Solution Statement 2 is incorrect. International Debt Statistics is published by the World Bank.

Statement 1 and 3 are correct.

External debt includes long term debt like NRI deposits and commercial borrowings.

* Note: Irrespective of what the latest figures are There remain a similar trend in Indian Foreign Trade. (Until there are adverse situation)

{i.e. Exports will usually increase. Imports will usually increase. Increase in Imports will be higher than that of Exports, Hence Current Account Deficit will exist. And India will remain a net exporter of Services}

These Trends changes only in adverse circumstances. UPSC is hardly concerned about figures but the can ask questions from Trends.

Ref: http://pib.nic.in/newsite/PrintRelease.aspx?relid=170822

8. 38 degree Lattitude (38th Parallel) Line seprates:

A. Japanese Archipelago with Yellow sea

B. North Korea and South Korea

C. USA and Canada

D. Morrocco and Spain

Correct Answer: B

Solution 38th Parallel is the Demilitarized Zone between North Korea and South Korea.

Ref: http://www.newindianexpress.com/opinions/2017/sep/15/a-korean-normandy-1657484.html (http://www.newindianexpress.com/opinions/2017/sep/15/a-korean-normandy-1657484.html) http://www.jigyasaias.com/Admin/printDailyTest/DTEST59BCFFC09E54E 4/6 9/25/2017 Print Question Paper

9. Which of the following statements is/are correct?

1. According to the constitution, a new States in India can only be created on the basis of distinct Language and/or Ethnicity. 2. Tamil Nadu was the first state to be formed on linguistic Basis.

Choose the correct option.

A. 1 only

B. 2 only

C. Both 1 and 2

D. Neither 1 nor 2

Correct Answer: D

Solution Both the statements are incorrect.

Constitution does not provide for any basis of the creation of states.It has given a free hand to the Parliament to form states the way it wishes.

Statement 2 is also incorrect,

Andhra Pradesh was the First State to be formed on the basis of Language.

Ref: M.Laxmikanth

10. Which of the following statements is/are correct about Snow Leopard:

1. Snow leopards live in the mountainous regions of central and southern Asia 2. In India, their geographical range encompasses a large part of the western Himalayas including the states of Jammu and Kashmir, Himachal Pradesh, Uttarakhand and Sikkim and Arunachal Pradesh in the eastern Himalayas.

3. They are Critically Endangered Animal.

Choose the correct option.

A. 1 only

B. 1 and 2 only

C. 2 and 3 only

D. All of the above

Correct Answer: B

Solution Statement 3 is incorrect.

It was earlier Endangered Animal but its condition has improved with the conversation efforts.

Statement 1 and 2 are correct. Snow leopards live in the mountainous regions of central and southern Asia. In India, their geographical range encompasses a large part of the western Himalayas including the states of Jammu and Kashmir, Himachal Pradesh, Uttarakhand and Sikkim and Arunachal Pradesh in the eastern Himalayas. The last three states form part of the Eastern Himalayas – a priority global region of WWF and the Living Himalayas Network Initiative.

http://www.jigyasaias.com/Admin/printDailyTest/DTEST59BCFFC09E54E 5/6 9/25/2017 Print Question Paper

Snow leopards prefer steep, rugged terrains with rocky outcrops and ravines. This type of habitat provides good cover and clear view to help them sneak up on their prey. They are found at elevations of 3,000-5,000 metres or higher in the Himalayas.

Ref: https://www.nytimes.com/2017/09/15/science/snow-leopard-endangered.html (https://www.nytimes.com/2017/09/15/science/snow-leopard-endangered.html)

http://www.jigyasaias.com/Admin/printDailyTest/DTEST59BCFFC09E54E 6/6 9/25/2017 Print Question Paper

1. World bank can provide long-term loan for:

1. The setting of Current Account Deficit. 2. To promote Foreign Direct Investment into developing countries to support economic growth. 3. Operations address primary education, basic health services, clean water and sanitation, environmental safeguards, business climate improvements.

Choose the correct options.

A. 1 and 2 only

B. 2 and 3 only

C. All of the above

D. None of the above

Correct Answer: B

Solution World Bank does not provide loans for setting off Current Account Deficit; that can be provided by IMF.

The World Bank Group has two ambitious goals:

End extreme poverty within a generation and boost shared prosperity.

It is a group of 5 organizations.

The International Bank for Reconstruction and Development The International Bank for Reconstruction and Development (IBRD) lends to governments of middle-income and creditworthy low-income countries.

The International Development Association The International Development Association (IDA) provides interest-free loans — called credits — and grants to governments of the poorest countries.

The International Finance CorporationThe International Finance Corporation (IFC) is the largest global development institution focused exclusively on the private sector. We help developing countries achieve sustainable growth by financing investment, mobilizing capital in international financial markets, and providing advisory services to businesses and governments.

The Multilateral Investment Guarantee Agency The Multilateral Investment Guarantee Agency (MIGA) was created in 1988 to promote foreign direct investment into developing countries to support economic growth, reduce poverty, and improve people‘s lives. MIGA fulfills this mandate by offering political risk insurance (guarantees) to investors and lenders.

The International Centre for Settlement of Investment Disputes The International Centre for Settlement of Investment Disputes (ICSID) provides international facilities for conciliation and arbitration of investment disputes.

Ref: http://www.worldbank.org/en/about (http://www.worldbank.org/en/about)

2. Sadhu Bet river island is located on which river?

A. Brahmaputra

B. Sabarmati

C. Narmada

D. Kaveri

Correct Answer: C

http://www.jigyasaias.com/Admin/printDailyTest/DTEST59BF86C02B341 1/6 9/25/2017 Print Question Paper

Solution Sadhu Bet is a river Island on Narmada river,

It is the island on which Statue of Unity (a statue of Sardar Vallabh Bhai Patel will be built)

Ref: http://economictimes.indiatimes.com/news/politics-and-nation/pm-narendra-modi-dedicates-sardar-sarovar-dam- on-river-narmada-to-the-nation/articleshow/60717518.cms

3. Which of the following statements is/are correct about North-East Monsoon?

1. In North-East monsoon winds blow parallel to Trade Winds. 2. It generally remains dry as compared to South-West Monsoon. 3. Tamil Nadu is the only state in India to receive rain from North-East Monsoon.

Choose the correct option.

A. 1 and 2 only

B. 2 only

C. 1 and 3 only

D. All of the above

Correct Answer: A

Solution Statement 1 is correct,

South-West monsoon starts when Inter-Tropical Convergence Zone (ITCZ) Shifts to Indian Sub-Continent which results into 'Seasonal Reversal of Trade Wind' and moisture-laden winds enter the Indian Subcontinent.

North-Eastern Monsoon is when the ITCZ moves back, which results in the wind blowing parallel to Trade Winds.

As these winds blow from land it usually remains dry.

The third statement is incorrect, Coastal Andhra Pradesh, Rayalaseema and Tamilnadu-Pondicherry, Interior Karnataka, Kerala and Lakshadweep etc. also receive rain from North-East monsoon.

Ref: http://www.imdchennai.gov.in/northeast_monsoon.htm (http://www.deccanchronicle.com/nation/current- affairs/100917/telangana-departments-gear-up-for-northeast-monsoon.html)

(http://www.deccanchronicle.com/nation/current-affairs/100917/telangana-

departments-gear-up-for-northeast-monsoon.html)

http://www.jigyasaias.com/Admin/printDailyTest/DTEST59BF86C02B341 2/6 9/25/2017 Print Question Paper

4. Which of the following statements is/are correct about Powers of Panchayati Raj Institutes?

1. 73rd Amendment Act in 1992, introduced Panchayati Raj Institutions for the first time in India. 2. It is in the hands of the State legislature to grant powers and authority to the panchayats. 3. One-third seats in panchayats at all the levels are reserved for Women in every state.

Choose the correct answer.

A. 1 and 2 only

B. 2 and 3 only

C. 1 and 3 only

D. All of the above

Correct Answer: B

Solution Statement 1 is incorrect, the Panchayati Raj Institutions have been in existence for a long time 73rd amendment just granted it a constitutional status.

Statement 2 and 3 are correct.

Ref: http://indiacode.nic.in/coiweb/amend/amend73.htm http://www.ptinews.com/news/9078066_PRIs-will-be-strengthened-further-- Khattar.html (http://indiacode.nic.in/coiweb/amend/amend73.htm)

5. Kalamkari Paintings are referred to:

A. A hand painting Cotton Textile in South India

B. A handmade drawing on Bamboo Handicrafts in North East India

C. A Block Painted Woolen Cloth of Western Himalayan Region

D. A Hand Made Decorative Silk Cloth in North-Western

India Correct Answer: A

Solution Kalamkari is a type of hand-painted or block-printed cotton textile, produced in Andhra Pradesh.

Ref: http://www.thehindu.com/news/national/andhra-pradesh/pigment-effluents-pose-threat-to-water- table/article19705200.ece (http://www.thehindu.com/news/national/andhra-pradesh/pigment-effluents-pose-threat-to- water-table/article19705200.ece)

6. Which of the following statements is/are correct about Krishna River?

1. It rises in the Western Ghats and Drains into the Arabian Sea. 2. Nagarjuna Sagar Dam is built across this river 3. It is the longest river of Peninsular India.

Choose the correct.

A. 1 and 2 only

B. 2 only http://www.jigyasaias.com/Admin/printDailyTest/DTEST59BF86C02B341 3/6 9/25/2017 Print Question Paper

C. 1 and 3 only

D. 2 and 3 only

Correct Answer: B

Solution Statement 1 is incorrect.

Krishna River rises in the Western Ghats and Drains into the Bay of Bengal.

Statement 3 is incorrect, the Godavari is the Longest River of Peninsular India.

Only Statement 2 is correct.

Ref: http://india-wris.nrsc.gov.in/wrpinfo/?title=Krishna http://www.thehindu.com/todays-paper/tp-national/tp-andhrapradesh/parched-prakasam-district-desperate-for-krishna- water/article19705919.ece (http://india-wris.nrsc.gov.in/wrpinfo/?title=Krishna)

7. Which of the following statements is/are correct?

1. In the case of absence of Chief Justice of the High Court, the President can appoint an Acting Chief Justice of the High Court. 2. The Chief Justice of a High Court can request a retired Judge of a High Court to act as a judge for a temporary period.

Choose the correct option.

A. 1 only

B. 2 only

C. Both 1 and 2

D. Neither 1 nor 2

Correct Answer: C

Solution Both the statements are correct.

Ref: M.Laxmikanth,

http://www.thehindu.com/news/national/no-regular-cjs-in-six-high-courts/article19704143.ece (http://www.thehindu.com/news/national/no-regular-cjs-in-six-high-courts/article19704143.ece)

8. Which of the following statements is/are correct regarding Shell Companies:

1. Shell companies are typically corporate entities which do not have any active business operations or significant assets in their possession. 2. The Companies Act, 2013 defines ‗shell companies‘ and the law governing it.

Choose the correct option.

A. 1 only

B. 2 only

C. Both 1 and 2 http://www.jigyasaias.com/Admin/printDailyTest/DTEST59BF86C02B341 4/6 9/25/2017 Print Question Paper

D. Neither 1 nor 2

Correct Answer: A

Solution Statement 2 is incorrect. The Companies Act, 2013 has not defined what a ‗shell company‘ is and as to what kind of activities would lead to a company being termed a ‗shell‘.

In India, there is no specific law relating to ―shell companies.‖ However, some laws help, to an extent, in curbing illegal activities such as money laundering and can indirectly be used to target shell companies — Benami Transaction (Prohibition) Amendment Act 2016; The Prevention of Money Laundering Act 2002 and The Companies Act, 2013.

Statement 1 is correct.

Ref: http://www.thehindu.com/business/Industry/decoding-shell-companies/article19704204.ece (http://www.thehindu.com/business/Industry/decoding-shell-companies/article19704204.ece)

9. Which of the following statements is/are correct about Sardar Sarovar Dam?

1. It is the biggest concrete gravity dam by volume in India. 2. It is fully built by World Bank's Funding. 3. It is an inter-state water project providing water to Madhya Pradesh, Gujarat, Rajasthan, and Maharashtra.

Choose the correct option.

A. 1 and 2 only

B. 2 and 3 only

C. 1 and 3 only

D. All of the above

Correct Answer: C

Solution Statement 2 is incorrect,

The World Bank refused to fund this project citing environmental damage, displacement of Tribal etc.

Statement 1 and 3 are correct.

Ref: http://www.thehindu.com/videos/narendra-modi-inaugurates-sardar-sarovar-dam/article19708621.ece (http://www.thehindu.com/videos/narendra-modi-inaugurates-sardar-sarovar-dam/article19708621.ece)

10. Antrix Corporation is a:

A. Joint Venture of India and Japan in Space Research

B. Corporation Arm of ISRO

C. PSU under Ministry of Science

D. None of the above

Correct Answer: B

Solution

http://www.jigyasaias.com/Admin/printDailyTest/DTEST59BF86C02B341 5/6 9/25/2017 Print Question Paper

Antrix Corporation Limited (Antrix), incorporated on 28 September 1992 (under the Companies Act, 1956), is a wholly owned Government of India Company under the administrative control of Department of Space (DOS). Antrix is the commercial arm of Indian Space Research Organisation (ISRO). Antrix promotes and commercially markets the products and services emanating from the Indian Space Programme. In the year 2008, the Company was awarded ‗MINIRATNA‘ status.

The current business activities of Antrix include:

a) Provisioning of communication satellite transponders to various users, b) Providing launch services for customer satellites, c) Marketing of data from Indian and foreign remote sensing satellites, d) Building and marketing of satellites as well as satellite sub-systems, e) Establishing ground infrastructure for space applications, and f) Mission support services for satellites.

Ref: http://www.thehindu.com/news/national/isro-ex-chief-madhavan-nair-summoned-as-accused-in-antrix-devas-deal- case/article19697960.ece (http://www.thehindu.com/news/national/isro-ex-chief-madhavan-nair-summoned-as-accused- in-antrix-devas-deal-case/article19697960.ece)

http://www.jigyasaias.com/Admin/printDailyTest/DTEST59BF86C02B341 6/6 9/25/2017 Print Question Paper

1. Which of the following statements is/are correct about 'Encephalitis'?

1. It is caused by Virus. 2. It can spread through contaminated water, insect bites etc. 3. Its transmission can intensify during the rainy seasons and pre-Harvest Period.

Choose the correct option.

A. 1 only

B. 2 and 3 only

C. 1 and 3 only

D. All of the above

Correct Answer: D

Solution Viral encephalitis is inflammation of the brain, caused by any one of a number of viruses. Symptoms include high fever, headache, sensitivity to light, stiff neck and back, vomiting, confusion and, in severe cases, seizures, paralysis, and coma. Infants and elderly people are particularly at risk of severe illness. Arboviruses – viruses transmitted through insect bites – are among the most common causes of viral encephalitis, and include Japanese encephalitis and tick- borne encephalitis viruses.

Its transmission can intensify during the rainy seasons and pre-Harvest Period.

Ref: http://www.who.int/topics/encephalitis_viral/en/ (http://www.who.int/topics/encephalitis_viral/en/) http://www.thehindu.com/todays-paper/tp-opinion/the-encephalitis-challenge/article19712084.ece (http://www.thehindu.com/todays-paper/tp-opinion/the-encephalitis-challenge/article19712084.ece)

2. Which of the following statements is/are correct?

1. Brown Feild Investments are related to investments in leather and meat production industries. 2. Green Feild investments are related to Investments in Food Production Industries.

Choose the correct option.

A. 1 only

B. 2 only

C. Neither 1 nor 2

D. Both 1 and 2

Correct Answer: C

Solution Both the statements are incorrect.

The Greenfield project means that a work which is not following a prior work. In infrastructure, the projects on the unused lands where there is no need to remodel or demolish an existing structure are called Green Field Projects. The projects which are modified or upgraded are called brownfield projects.

Ref: http://pib.nic.in/newsite/PrintRelease.aspx?relid=170895 (http://pib.nic.in/newsite/PrintRelease.aspx?relid=170895)

3. (Basic Questions...... for Beginners) Great Barrier Reef is located near the Coast of?

http://www.jigyasaias.com/Admin/printDailyTest/DTEST59C0CF7C3574A 1/6 9/25/2017 Print Question Paper

A. Canada

B. Australia

C. Madagascar

D. Brazil

Correct Answer: B

Solution The Great Barrier Reef is the world's largest coral reef system composed of over 2,900 individual reefs and 900 islands stretching for over 2,300 kilometers (1,400 mi) over an area of approximately 344,400 square kilometers (133,000 sq mi). The reef is located in the Coral Sea, off the coast of Queensland, Australia.

The Great Barrier Reef is blessed with the breathtaking beauty of the world‘s largest coral reef. The reef contains an abundance of marine life and comprises of over 3000 individual reef systems and coral cays and literally hundreds of picturesque tropical islands with some of the worlds most beautiful sun-soaked, golden beaches.Because of its natural beauty, the Great Barrier Reef has become one of the worlds most sought after tourist destinations.

Ref: http://www.thehindu.com/sci-tech/energy-and-environment/giant-sea-snail-plan-to-rescue-barrier- reef/article19710156.ece (http://www.thehindu.com/sci-tech/energy-and-environment/giant-sea-snail-plan-to-rescue- barrier-reef/article19710156.ece)

4. (Basic Questions...... For Beginners) Which of the following is a West Flowing Peninsular River?

A. Palar

B. Pennar

C. Periyar

D. Vaigai

Correct Answer: C

Solution Palar, Pennar and Vaigai are rivers of South India that flow from west towards east and merges with Bay of Bengal.

Periyar (meaning: big river) is the longest river and the river with the largest discharge potential in the Indian state of Kerala, it flows west wards.

http://www.jigyasaias.com/Admin/printDailyTest/DTEST59C0CF7C3574A 2/6 9/25/2017 Print Question Paper

Ref: http://www.thehindu.com/todays-paper/tp-national/tp-kerala/after-downpour-its-raining-miseries-for- kochiites/article19712924.ece (http://www.thehindu.com/todays-paper/tp-national/tp-kerala/after-downpour-its-raining- miseries-for-kochiites/article19712924.ece) http://www.india-wris.nrsc.gov.in/wrpinfo/index.php?title=Periyar (http://www.india-wris.nrsc.gov.in/wrpinfo/index.php? title=Periyar)

5. Which of the following statements is/are correct about Disqualification of Members of Legislative Assembly?

1. Only the Governor has the power to declare an MLA disqualified on the Ground of Defection. 2. Governor should obtain the opinion of Election Commission before disqualifying an MLA on the grounds of Violation of Representation of People's Act.

Choose the correct answer.

A. 1 only

B. 2 only

C. Both 1 and 2

D. Neither 1 nor 2

Correct Answer: B

Solution Statement 1 is incorrect;

According to the Tenth Schedule of the constitution, it is the power of the Preceding officer of the house (in this case the Speaker of the Legislative Assembly) to disqualify a member of the grounds of Defection.

Statement 2 is correct.

Ref: M.Laxmikanth http://www.hindustantimes.com/india-news/tamil-nadu-speaker-disqualifies-18-dinakaran-faction-mlas-palaniswami-govt- safe/story-HkWmYeTHIjFlyX2ud58vnK.html (http://www.hindustantimes.com/india-news/tamil-nadu-speaker-disqualifies-18- dinakaran-faction-mlas-palaniswami-govt-safe/story-HkWmYeTHIjFlyX2ud58vnK.html)

6. Which of the following statements is/are correct about GST Cess?

1. The proceeds of the cess will be distributed to loss-incurring States on the basis of a prescribed formula. 2. According to GST Act, even after implementation of GST Cess the total tax on a product cannot be more than 28% of its Market Price.

Choose the correct option

A. 1 only

B. 2 only

C. Both 1 and 2

D. Neither 1 nor 2

Correct Answer: A

Solution Statement 2 is incorrect,

Though GST rate cannot extend more than 28% of the market price, but the GST cess can be levied over and above that value. http://www.jigyasaias.com/Admin/printDailyTest/DTEST59C0CF7C3574A 3/6 9/25/2017 Print Question Paper

Statement 1 is correct,

Ref: http://www.thehindu.com/opinion/op-ed/questions-about-the-gst-cess/article19710552.ece http://www.cbec.gov.in/resources//htdocs-cbec/gst/notfctn-1-compensation-cess-english.pdf (http://www.thehindu.com/opinion/op-ed/questions-about-the-gst-cess/article19710552.ece)

7. Which of the following statements is/are correct about Coral Reefs:

1. Polar regions produce ideal climate for Coral Development 2. Sedimentation and Erosion can 'smother' corals by depriving them of the light needed to survive. 3. Apart from anthropogenic causes the corals also face threats from natural predators.

Choose the correct answer.

A. 1 and 2 only

B. 2 and 3 only

C. 1 and 3 only

D. All of the above

Correct Answer: B

Solution Statement 1 is incorrect,

Coral Reefs cannot survive if the temperature is too hot or too cold, hence Polar regions are not ideal for their survival, in fact, it does not exist above tropics.

Statement 2 and 3 are correct,

Ref: http://www.thehindu.com/sci-tech/energy-and-environment/giant-sea-snail-plan-to-rescue-barrier- reef/article19710156.ece (http://www.thehindu.com/sci-tech/energy-and-environment/giant-sea-snail-plan-to-rescue-barrier- reef/article19710156.ece) http://wwf.panda.org/about_our_earth/blue_planet/coasts/coral_reefs/coral_threats/ (http://wwf.panda.org/about_our_earth/blue_planet/coasts/coral_reefs/coral_threats/)

8. Which of the following statements is/are correct?

1. A cess is a levy for a specific purpose While A surcharge may not be included for the stated purpose. 2. A Cess is a certain percentage of a tax as an additional tax, while surcharge on tax is a fixed sum collected for a specific reason.

Choose the correct option.

A. 1 only

B. 2 only

C. Both 1 and 2

D. Neither 1 nor 2

Correct Answer: A

Solution Statement 2 is incorrect,

http://www.jigyasaias.com/Admin/printDailyTest/DTEST59C0CF7C3574A 4/6 9/25/2017 Print Question Paper

Surcharge' is an additional charge or tax levied on an existing tax. Unlike a cess, which is meant to raise revenue for a temporary need, the surcharge is usually permanent in nature. It is levied as a percentage of the income tax payable as per normal rates.

Ref: http://www.thehindu.com/opinion/op-ed/questions-about-the-gst-cess/article19710552.ece

9. Which of the following ethnicity is/are correctly matched with their Countries?

1. Madhesis: Afganistan. 2. Rohingyas: Myanmar 3. Sinhalese: Sri Lanka 4. Chakma: Bhutan

Choose the correct option.

A. 1, 2 and 4 only

B. 2 and 3 only

C. 1 and 4 only

D. All of the above

Correct Answer: B

Solution Madhesis are the people of Nepalis and Chakmas were original dissidents of Bangladesh.

Other two are correctly Matched

Ref: http://indianexpress.com/article/explained/who-are-the-madhesis-why-are-they-angry/ (http://indianexpress.com/article/explained/who-are-the-madhesis-why-are-they-angry/)

http://www.hindustantimes.com/india-news/arunachal-shuts-down-over-citizenship-to-chakma-hajong-refugees/story- 5Zl1HYedOB2MkrhG85LEUK.html (http://www.hindustantimes.com/india-news/arunachal-shuts-down-over-citizenship- to-chakma-hajong-refugees/story-5Zl1HYedOB2MkrhG85LEUK.html)

10. Term 'Shell Companies' often remain in news. (Basic Question for Beginners) It is referred to:

A. Corporations engaged in Sea Shore Exploration

B. A non-trading company used as a vehicle for various financial maneuvers.

C. FDI based Defence corporations

D. Corporations with outstanding bank loans

Correct Answer: B

Solution Shell Companies are a non-trading company used as a vehicle for various financial maneuvers or kept dormant for future use in some other capacity.

Or, a corporation without active business operations or significant assets. These types of corporations are not all necessarily illegal, but they are sometimes used illegitimately, such as to disguise business ownership from law enforcement or the public. Legitimate reasons for a shell corporation include such things as a startup using the business entity as a vehicle to raise, funds, conduct a hostile takeover or to go public.

Ref: http://www.thehindu.com/todays-paper/tp-business/decoding-shell-companies/article19705833.ece (http://www.thehindu.com/todays-paper/tp-business/decoding-shell-companies/article19705833.ece) http://www.livemint.com/Money/gFqqUTa6pYGWVkjyHh58QO/What-are-shell-companies.html http://www.jigyasaias.com/Admin/printDailyTest/DTEST59C0CF7C3574A 5/6 9/25/2017 Print Question Paper

(http://www.livemint.com/Money/gFqqUTa6pYGWVkjyHh58QO/What-are-shell-companies.html)

http://www.jigyasaias.com/Admin/printDailyTest/DTEST59C0CF7C3574A 6/6 9/25/2017 Print Question Paper

1. Which of the following statements is/are correct about Election Disputes:

1. Disputes relating to Election of Members of State Legislature is Under the Original Jurisdiction of High Court. 2. Disputes relating to Election of Members of Parliament is Under the Original Jurisdiction of Supreme Court. 3. Disputes in connection with the Election of President is under the Original Jurisdiction of Supreme Court.

Choose the correct answer.

A. 1 and 2 only

B. 1 and 3 only

C. 2 and 3 only

D. All of the above

Correct Answer: B

Solution Original Jurisdiction is the power of the Court to hear disputes at the first instance, not by the way of Appeal.

Among the options, statement 2 is incorrect, as even disputes related to the election of Member of Parliament is under the original jurisdiction of High Court.

Ref: M.Laxmikanth

http://www.deccanchronicle.com/nation/current-affairs/200917/madras-hc-to-hear-18-dhinakaran-faction-mlas-plea- against-disqualification.html (http://www.deccanchronicle.com/nation/current-affairs/200917/madras-hc-to-hear-18- dhinakaran-faction-mlas-plea-against-disqualification.html)

2. Which of the following statements is/are correct about Alpana Art:

1. It is a form of Rangoli practiced in Bengal. 2. It is intrinsically linked with religious austerity practiced by women. 3. It has got the status of a Geographical indicator.

Choose the correct option.

A. 1 and 2 only

B. 2 and 3 only

C. All of the above

D. None of the Above

Correct Answer: A

Solution Alpana, the form of Rangoli practiced in Bengal, is a natural representation of the artistic sensibility of the people. Practiced usually by the womenfolk of the state, the art form represents an amalgamation of the past experience as well as the contemporary designs. Even though the basic designs are more or less same, new forms and new colors are being tried on a large scale. The changing moods of the seasons are also very much reflected in the Alpana designs of India. The patterns are made with the help of a small piece of cloth drenched in a blend of powdered rice.

It does not have any GI status.

Ref: http://www.thehindu.com/news/national/other-states/Reviving-a-vanishing-folk-art-form-in-Bengal/article16083431.ece (http://www.thehindu.com/news/national/other-states/Reviving-a-vanishing-folk-art-form-in-Bengal/article16083431.ece) http://ipindia.nic.in/writereaddata/Portal/Images/pdf/Registered-GIs-of-India.pdf (http://indianexpress.com/article/lifestyle/art-and- culture/durga-puja-2017-kolkata-streets-decorated-with-longest-alpona-4851324/) http://www.jigyasaias.com/Admin/printDailyTest/DTEST59C228673FA56 1/5 9/25/2017 Print Question Paper

3. (Basic Question for Beginners) Kurdish Ethenic Group belongs to which region?

A. North African

B. Latin America

C. Middle East Asia

D. North America

Correct Answer: C

Solution Kurdish people belong to the Middle East region.

Ref: http://www.thehindu.com/opinion/editorial/the-kurdish-vote/article19715950.ece (http://www.thehindu.com/opinion/editorial/the-kurdish-vote/article19715950.ece)

4. Which of the following statements is/are correct about National Investigation Agency?

1. The NIA aims to set the standards of excellence in investigating tax evasion, Money Laundering, and Unaccounted Foreign Assets etc. 2. Ministry of Finance is the Nodal Agency for managing the NIA.

Choose the correct option

A. 1 only

B. 2 only

C. Both 1 and 2

D. Neither 1 nor 2

Correct Answer: D

Solution Both the statements are incorrect;

The National Investigation Agency aims to be a thoroughly professional investigative agency matching the best international standards. The NIA aims to set the standards of excellence in counter-terrorism and other national security- related investigations at the national level by developing a highly trained, partnership-oriented workforce. NIA aims at creating deterrence for existing and potential terrorist groups/individuals. It aims to develop as a storehouse of all terrorist- related information. http://www.jigyasaias.com/Admin/printDailyTest/DTEST59C228673FA56 2/5 9/25/2017 Print Question Paper

Though for terror-related cases it can also look for the source of its funding, but Primary task of NIA is counter- terrorism only.

Hence, it comes under Ministry of Home Affairs.

Ref: http://nia.gov.in/vision-mission.htm http://lawmin.nic.in/ld/P-ACT/2008/The%20National%20Investigation%20Agency%20Act,%202008.pdf (http://lawmin.nic.in/ld/P-ACT/2008/The%20National%20Investigation%20Agency%20Act,%202008.pdf) http://pib.nic.in/newsite/PrintRelease.aspx?relid=170881 (http://pib.nic.in/newsite/PrintRelease.aspx?relid=170881)

5. Which of the following statements is/are correct about Coal Bed Methane?

1. It is an unconventional form of natural gas found in coal deposits or coal seams. 2. Because of large coal reserves, there is also a possibility of large Coal bed methane reserve in India.

Choose the correct option.

A. 1 only

B. 2 only

C. Both 1 and 2

D. Neither 1 nor 2

Correct Answer: C

Solution Both the statements are correct.

Ref: http://www.thehindu.com/business/Economy/new-policy-may-help-revive-interest-in-coal-bed- methane/article19715816.ece (http://www.thehindu.com/business/Economy/new-policy-may-help-revive-interest-in- coal-bed-methane/article19715816.ece)

6. Which of the following statements is/are correct about United Nation Conference on Trade and Development (UNCTAD)?

1. UNCTAD is a permanent intergovernmental body. 2. UNCTAD is part of the World Trade Organisation. 3. It aims to support developing countries to access the benefits of a globalized economy more fairly and effectively.

Choose the correct option.

A. 1 only

B. 2 and 3 only

C. 1 and 2 only

D. 1 and 3 only

Correct Answer: D

Solution Statement 2 is incorrect, UNCTAD is part of the UN Secretariat. It reports to the UN General Assembly and the Economic and Social Council but has its own membership, leadership, and budget. Though, the WTO and UNCTAD are parties in a strategic partnership for the purpose of implementing the DDA, cooperating to ensure that trade serves development goals, and for assisting the beneficial integration of the developing and least developed countries (LDCs) into the global economy and the multilateral trading system. But it is not a part of WTO organizational Structure.

Statement 1 and 2 are absolutely correct.

http://www.jigyasaias.com/Admin/printDailyTest/DTEST59C228673FA56 3/5 9/25/2017 Print Question Paper

Ref: http://unctad.org/en/Pages/aboutus.aspx https://www.wto.org/english/thewto_e/coher_e/wto_unctad_e.htm (http://unctad.org/en/Pages/aboutus.aspx)

7. (Basic Question for Beginners) Austerity Measures are referred to:

A. Actions taken by the government to reduce its Fiscal deficit

B. Increasing Government Spending.

C. Opening Domestic Economy for foreign Investments

D. Deregulation of Industries.

Correct Answer: A

Solution Austerity measures refer to official actions taken by the government, during a period of adverse economic conditions, to reduce its budget deficit using a combination of spending cuts or tax rises.

Opening Domestic Economy for foreign investments is globalization and Deregulation of Industries are called Liberalisation, though they can bring austerity but not necessarily, hence Option A is the right answer.

Ref: http://www.thehindu.com/business/Economy/india-china-unlikely-to-be-growth-poles-for-global-economy- unctad/article19685905.ece (http://www.thehindu.com/business/Economy/india-china-unlikely-to-be-growth-poles-for- global-economy-unctad/article19685905.ece)

8. Puthuvypee Island is located in which state?

A. Karnataka

B. Kerala

C. Tamil Nadu

D. Andhra Pradesh

Correct Answer: B

Solution The expert committee appointed by the government to study the environmental and safety aspects of the Indian Oil Corporation‘s (IOC) LPG import terminal at Puthuvype will hold its second meeting shortly before undertaking the site visit to Puthuvype island and meeting all stakeholders.

Ref http://www.newindianexpress.com/cities/kochi/2017/sep/12/ioc-plant-at-puthuvype-committee-to-submit-report-next- month-end-1655694.html (http://www.newindianexpress.com/cities/kochi/2017/sep/12/ioc-plant-at-puthuvype-committee- to-submit-report-next-month-end-1655694.html)

9. Which of the following statements is/are correct about Hague Convention on International Child Abduction?

1. Hague Convention on International Child Abduction rules that in any child custody case, the court of the country where the child is a ―habitual resident‖ will adjudicate who will get custody. 2. India is a signatory to this convention.

Choose the correct answer.

http://www.jigyasaias.com/Admin/printDailyTest/DTEST59C228673FA56 4/5 9/25/2017 Print Question Paper

A. 1 only

B. 2 only

C. Both 1 and 2

D. Neither 1 nor 2

Correct Answer: A

Solution Statement 2 is incorrect, India is not a signatory to this convention, In Nov, 2016 India had decided not to sign it, while government is thinking about Reconsidering its decision.

Statement 1 is correct.

Ref: http://www.thehindu.com/news/national/centre-rethinks-joining-hague-child-custody-pact/article19716400.ece (http://www.thehindu.com/news/national/centre-rethinks-joining-hague-child-custody-pact/article19716400.ece)

10. Which of the following statements is/are correct about North Korea?

1. It is not a member country of UNO. 2. It is a Nuclear Power Country. 3. It is a Land Locked Country.

Choose the correct option.

A. 1 and 2 only

B. 2 only

C. 1 and 3 only

D. All of the above

Correct Answer: B

Solution Statement 1 is incorrect, North Korea is a member of United Nation since 1991. Ref: http://www.un.org/en/member-states/#gotoN (http://www.un.org/en/member-states/#gotoN)

Statement 3 is incorrect it is not landlocked it does have sea frontier.

Only statement 2 is correct.

http://www.jigyasaias.com/Admin/printDailyTest/DTEST59C228673FA56 5/5 9/25/2017 Print Question Paper

1. Which of the following statements is/are correct about Sustainable Development Goals?

1. They were adopted by world leaders at a UN Summit. 2. It is a set of 30 goals targeted to be achieved by 2017. 3. It is legally binding on the members.

Choose the correct answer.

A. 1 only

B. 2 and 3 only

C. 1 and 2 only

D. All of the above

Correct Answer: A

Solution Statement 1 is correct.

17 Sustainable Development Goals (SDGs) of the 2030 Agenda for Sustainable Development — adopted by world leaders in September 2015 at a historic UN Summit — officially came into force. Over the next fifteen years, with these new Goals that universally apply to all, countries will mobilize efforts to end all forms of poverty, fight inequalities and tackle climate change, while ensuring that no one is left behind.

Statement 2 and 3 are incorrect, it is a set of 17 goals to be achieved by 2030.

and it is not legally binding on anyone.

Ref: http://www.un.org/sustainabledevelopment/development-agenda/ (http://www.un.org/sustainabledevelopment/development-agenda/)

2. Which of the following statements is/are correct about 'Equinox?

1. On Equinox the sun's rays fall perpendicular on The Equator. 2. This day witness the longest day in Northern Hemisphere and Shortest Day on Southern Hemisphere.

Choose the correct option.

A. 1 only

B. 2 only

C. Both 1 and 2

D. Neither 1 nor 2

Correct Answer: A

Solution Statement 2 is incorrect

There are two equinoxes every year – in September and March – when the sun shines directly on the equator and the length of day and night is nearly equal.

Seasons are opposite on either side of the Equator, so the equinox in September is also known as the autumnal (fall) equinox in the Northern Hemisphere, and is considered the first day of fall.

In the Southern Hemisphere, it is known as the vernal (spring) equinox and marks the first day of spring. Ref: G.C.Leong

http://www.jigyasaias.com/Admin/printDailyTest/DTEST59C375D239D5C 1/6 9/25/2017 Print Question Paper

3. Which of the following statements is/are correct about GST Refunds?

1. Firms can claim transitional credits for inputs bought and taxes paid before GST rollout. 2. A claim for refund may arise on account of Export of goods or services, Supplies to SEZs units and developers, Refund of taxes on purchase made by UN or embassies etc.

Choose the correct answer.

A. 1 only

B. 2 only

C. Both 1 and 2

D. Neither 1 nor 2

Correct Answer: C

Solution Both the statements are correct,

The relevant date provision embodied in Section 54 of the CGST Act, 2017, provision contained in Section 77 of the CGST Act, 2017 and the requirement of submission of relevant documents as listed in Rule 1(2) of Refund Rules is an indicator of the various situations that may necessitate a refund claim. A claim for refund may arise on account of: 1. Export of goods or services 2. Supplies to SEZs units and developers 3. Deemed exports 4. Refund of taxes on purchase made by UN or embassies etc. 5. Refund arising on account of judgment, decree, order or direction of the Appellate Authority, Appellate Tribunal or any court 6. Refund of accumulated Input Tax Credit on account of inverted duty structure 7. Finalisation of provisional assessment 8. Refund of pre-deposit 9. Excess payment due to mistake. etc.

Ref: http://www.thehindu.com/business/Economy/chunk-of-gst-claims-ineligible-for-refunds-finance- ministry/article19723131.ece (http://www.thehindu.com/business/Economy/chunk-of-gst-claims-ineligible-for-refunds- finance-ministry/article19723131.ece) http://www.cbec.gov.in/resources//htdocs-cbec/gst/eflier- refunds14062017.pdf;jsessionid=2D97A1F3B5831D6FCB09194043D9B584 (http://www.cbec.gov.in/resources//htdocs- cbec/gst/eflier-refunds14062017.pdf;jsessionid=2D97A1F3B5831D6FCB09194043D9B584)

http://www.jigyasaias.com/Admin/printDailyTest/DTEST59C375D239D5C 2/6 9/25/2017 Print Question Paper

4. Which of the following statements is/are correct about Padma Bhushan?

1. Government servants including those working with PSUs, except doctors and scientists, are not eligible for these Awards. 2. The award does not amount to a title and cannot be used as a suffix or prefix to the awardees‘ name. 3. The award seeks to recognize works of distinction in the fields of Art, Social work, Public Affairs, Science & Engineering, Trade & Industry, Medicine, Literature & Education, Civil Service, Sports etc.

Choose the correct option

A. 1 and 2 only

B. 2 and 3 only

C. 1 and 3 only

D. All of the above

Correct Answer: D

Solution All the statements are correct.

All persons without distinction of race, occupation, position or sex are eligible for these awards. However, Government servants including those working with PSUs, except doctors and scientists, are not eligible for these Awards.

The award seeks to recognize works of distinction and is given for distinguished and exceptional achievements/service in all fields of activities/disciplines. An illustrative list of the fields is as under:

Art (includes Music, Painting, Sculpture, Photography, Cinema, Theatre etc.)

Social work (includes social service, charitable service, contribution in community projects etc.)

Public Affairs (includes Law, Public Life, Politics etc.)

Science & Engineering (includes Space Engineering, Nuclear Science, Information Technology, Research & Development in Science & its allied subjects etc.)

Trade & Industry (includes Banking, Economic Activities, Management, Promotion of Tourism, Business etc.)

Medicine (includes medical research, distinction/specialization in Ayurveda, Homeopathy, Sidhha, Allopathy, Naturopathy etc.)

Literature & Education (includes Journalism, Teaching, Book composing, Literature, Poetry, Promotion of education, Promotion of literacy, Education Reforms etc.)

Civil Service (includes distinction/excellence in administration etc. by Government Servants)

Sports (includes popular Sports, Athletics, Adventure, Mountaineering, promotion of sports, Yoga etc.)

Others (fields not covered above and may include propagation of Indian Culture, protection of Human Rights, Wild Life protection/conservation etc.)

The award is normally not conferred posthumously. However, in highly deserving cases, the Government could consider giving an award posthumously.

A higher category of Padma award can be conferred on a person only where a period of at least five years has elapsed since conferment of the earlier Padma award. However, in highly deserving cases, a relaxation can be made by the Awards Committee.

The awards are presented by the President of India usually in the month of March/April every year where the awardees are presented a Sanad (certificate) signed by the President and a medallion.

The recipients are also given a small replica of the medallion, which they can wear during any ceremonial/State functions etc., if the awardees so desire. The names of the awardees are published in the Gazette of India on the day of the presentation ceremony.

The total number of awards to be given in a year (excluding posthumous awards and to NRI/foreigners/OCIs) should not be more than 120.

The award does not amount to a title and cannot be used as a suffix or prefix to the awardees‘ name http://www.jigyasaias.com/Admin/printDailyTest/DTEST59C375D239D5C 3/6 9/25/2017 Print Question Paper

5. (Basic Question for the Beginers)

Traffic in human beings and forced labour is prohibited in India By which article of the constitution?

A. Article 14

B. Article 23

C. Article 29

D. Article 35

Correct Answer: B

Solution Article 23 in The Constitution Of India states:

Prohibition of traffic in human beings and forced labour

(1) Traffic in human beings and begar and other similar forms of forced labour are prohibited and any contravention of this provision shall be an offence punishable in accordance with law

However

(2) Nothing in this article shall prevent the State from imposing compulsory service for the public purpose, and in imposing such service the State shall not make any discrimination on grounds only of religion, race, caste or class or any of them.

6. Which of the following statements is/are correct about Cell for IPR Promotions & Management (CIPAM).

1. It is a cell under the aegis of Department of Industrial Policy and Promotion (DIPP), Ministry of Commerce and Industry. 2. It is the institutional mechanism through which the National Intellectual Property Rights (IPR) Policy would be implemented.

Choose the correct option

A. 1 only

B. 2 only

C. Neither 1 nor 2

D. Both 1 and 2

Correct Answer: D

Solution Both the statements are correct.

Cell for IPR Promotion and Management (CIPAM) is the institutional mechanism through which the National Intellectual Property Rights (IPR) Policy would be implemented. CIPAM is a professional body under the aegis of Department of Industrial Policy and Promotion, Ministry of Commerce and Industry, Government of India. CIPAM will facilitate promotion, creation and commercialization of IP assets.

Ref: http://www.thehindu.com/opinion/op-ed/beyond-social-media/article19723042.ece (http://www.thehindu.com/opinion/op-ed/beyond-social-media/article19723042.ece)

http://www.jigyasaias.com/Admin/printDailyTest/DTEST59C375D239D5C 4/6 9/25/2017 Print Question Paper

7. Which of the following statements is/are correct?

1. Carbon Monoxide in High Concentration reduces oxygen supply to the vital organs. 2. Ground Level Ozone is helpful in protecting plants from harmful ultraviolate rays. 3. Nitrozen Oxide aggravates respiratory illness, reacting with other particles it can also cause Acid Rain.

Choose the correct option.

A. 1 only

B. 1 and 3 only

C. 2 and 3 only

D. All of the above

Correct Answer: B

Solution Statement 2 is incorrect,

While Stratospheric Ozone (higher level ozone or 'good' ozone) protect the environment by reflecting back the harmful ultraviolet rays.

Ozone at the lower level (Bad) is harmful to health, it can trigger a variety of health problems, particularly for children, the elderly, and people of all ages who have lung diseases such as asthma. Ground level ozone can also have harmful effects on sensitive vegetation and ecosystems.

Statement 1 and 3 are correct.

Ref: https://www.epa.gov/ozone-pollution (https://www.epa.gov/ozone-pollution)

The Hindu; weather watch

8. Which of the following statsments is/are correct about Chahbahar Port?

1. It is located in the Sistan-Balochistan province of Iran on the energy-rich Persian Gulf. 2. It will provide strategic connectivity to India with Afghanistan, Russia and Central Asia.

Choose the correct option

A. 1 only

B. 2 only

C. Both 1 and 2

D. Neither 1 nor 2

Correct Answer: C

Solution Both the statements are absolutely correct.

Located in the Sistan-Baluchistan province on the southeastern coast of Iran, Chabahar will provide strategic connectivity to India with Afghanistan, Russia and Central Asia. Located some 1000 km from Kandla and 1,400 km from Mumbai, Chabahar will ease sea route access for cargo, trade and business. India is developing and equipping two terminals at the Shahid Behshti-Chabahar port, which will be operated and maintained at a cost of nearly Rs 800 crore ($121 million) for 10 years.

Ref: http://www.tribuneindia.com/news/world/chabahar-project-to-be-ready-by-2018-gadkari/470610.html (http://www.tribuneindia.com/news/world/chabahar-project-to-be-ready-by-2018-gadkari/470610.html)

http://www.jigyasaias.com/Admin/printDailyTest/DTEST59C375D239D5C 5/6 9/25/2017 Print Question Paper

9. Chardham all-weather Highway project has recently got clearance by NGT.

This project of Union Ministry of Road Transport & Highways is related to which state?

A. Uttar Pradesh

B. Himachal Pradesh

C. Uttarakhand

D. Jammu and Kashmir

Correct Answer: C

Solution The Government has decided to upgrade highway connectivity to two lane with paved shoulders to Chardham through old National Highways no. 58, 94, 108 & 109 from Rishikesh to Kedarnath, Badrinath, Gangotri & Yamunotri and Tanakpur - Pithoragarh section of National Highways no. 125 at an estimated cost of about Rs. 11700 Crore including the cost of land acquisition and pre-construction activities targeted for completion by 2020. This also has provision for landslide mitigation at Kaliasaur ,Sonprayag, Maithana, Govindghat, Birahi,etc alongwith paved shoulders and other road safety measures, which will provide all weather road connectivity for pilgrims and for movement of defence forces.

Ref: http://economictimes.indiatimes.com/industry/transportation/roadways/decks-cleared-for-chardham-highway-project-ngt- disposes-plea/articleshow/60750665.cms (http://economictimes.indiatimes.com/industry/transportation/roadways/decks-cleared- for-chardham-highway-project-ngt-disposes-plea/articleshow/60750665.cms) http://pib.nic.in/newsite/mbErel.aspx?relid=158099 (http://pib.nic.in/newsite/mbErel.aspx?relid=158099)

10. Which of the following statements is/are correct about Quantum Computer:

1. It employs the principles of quantum mechanics to store information in ‗qubits‘ instead of the typical ‗bits‘ of 1 and 0. 2. Indian Department of Science and Technology (DST) is planning to fund a project to develop quantum computers. 3. India's PARAM 8000 (made by C-DAC) launched on July 01, 1991 was the firet quantum Computer developed in India.

Choose the correct option.

A. 1 and 2 only

B. 3 only

C. 1 and 3 only

D. All of the above

Correct Answer: A

Solution Statement 3 is incorrect, PARAM was a super Computer, Quantum Computers uses different technology than any other computers. (using quantum mechanics instead of Binary System)

Internationally, Canada‘s D-Wave Systems is a pioneer in developing quantum computers and has sold machines to Lockheed Martin and Google.

India is yet to develop its first Super Computer.

Ref: http://www.thehindu.com/news/national/india-joins-quantum-computing-race/article19723359.ece (http://www.thehindu.com/news/national/india-joins-quantum-computing-race/article19723359.ece)

http://www.jigyasaias.com/Admin/printDailyTest/DTEST59C375D239D5C 6/6 9/25/2017 Print Question Paper

1. Which of the following statements is/are correct about Shanghai Cooperation Organisation (SCO):

1. It is an Eight Country Block in which India, Pakistan, China, and Russia are members. 2. It is a Military Block of South Asian and Central Asian Countries to combat terrorism.

Choose the correct option.

A. 1 only

B. 2 only

C. Both 1 and 2

D. Neither 1 nor 2

Correct Answer: A

Solution Statement 1 is correct, the SCO comprises eight member states, namely, India, Kazakhstan, China, Kyrgyzstan, Pakistan, Russia, Tajikistan, and Uzbekistan;

Statement 2 is incorrect, It is not a military block.

The SCO's main goals are as follows: strengthening mutual trust and neighbourliness among the member states; promoting their effective cooperation in politics, trade, the economy, research, technology and culture, as well as in education, energy, transport, tourism, environmental protection, and other areas; making joint efforts to maintain and ensure peace, security and stability in the region; and moving towards the establishment of a democratic, fair and rational new international political and economic order.

Ref: http://eng.sectsco.org/about_sco/ http://www.ddinews.gov.in/national/terror-cannot-be-justified-india-makes-it-clear-sco-meet (http://www.ddinews.gov.in/national/terror-cannot-be-justified-india-makes-it-clear-sco-meet)

2. G4 countries are the group of four countries calling for an "early reform" of the UN Security Council and the enhanced role for developing countries in the UN.

Which of the following country is not a part of this Group?

A. India

B. Italy

C. Brazil

D. Japan

Correct Answer: B

Solution G4 countries - Brazil, Germany, India and Japan which support each other's bids for permanent seats on the United Nations Security Council.

Recently, At a meeting here on the sidelines of the annual General Assembly Sessions of the United Nations, A joint statement was issued after the meeting attended by External Affairs Minister Sushma Swaraj, Brazilian Foreign Relations Minister Aloysio Nunes, German Foreign Minister Sigmar Gabriel and Japanese Foreign Minister Tarp Kono.

The four countries called for an "early reform" of the UN Security Council and enhanced role for developing countries as major contributors to the UN and improvement of working methods in order to make the Council more legitimate, effective and representation

Ref: http://economictimes.indiatimes.com/news/defence/g4-countries-seek-early-reform-of-un-security- council/articleshow/60776007.cms (http://economictimes.indiatimes.com/news/defence/g4-countries-seek-early-reform- of-un-security-council/articleshow/60776007.cms) http://www.jigyasaias.com/Admin/printDailyTest/DTEST59C4E063D0E4D 1/6 9/25/2017 Print Question Paper

3. In which of the following sectors there is 100% Foreign Direct Investment permissable in India?

1. Textile 2. Food Processing 3. Defence Equippment

Choose the correct option.

A. 1 and 2 only

B. 2 and 3 only

C. 1 only

D. All of the above

Correct Answer: A

Solution India is a promising destination for FDI in the textile sector. 100% FDI is allowed in the textile sector under the automatic route. Ref: http://texmin.nic.in/fdi-cell (http://texmin.nic.in/fdi-cell)

In Food Processing Industry also there is 100% FDI available Ref: http://pib.nic.in/newsite/PrintRelease.aspx?relid=137944 (http://pib.nic.in/newsite/PrintRelease.aspx?relid=137944)

However India is still not open for 100% FDI in Defence as of yet, But there are indication that soon it will be opened.

Ref: http://www.thehindubusinessline.com/economy/policy/govt-may-allow-100-fdi-in-defence/article9824304.ece (http://www.thehindubusinessline.com/economy/policy/govt-may-allow-100-fdi-in-defence/article9824304.ece) http://economictimes.indiatimes.com/news/defence/india-open-to-100-per-cent-fdi-in-defence-with-full-tech- transfer/articleshow/60739657.cms (http://economictimes.indiatimes.com/news/defence/india-open-to-100-per-cent-fdi- in-defence-with-full-tech-transfer/articleshow/60739657.cms)

4. Which of the following statements is/are correct about ?

1. It is a UNESCO declared Natural world Heritage Site. 2. Agoratoli Eco Tourism Range, famous for migratory Birds is a part of this resort. 3. River Brahmputra passes through this National Park.

Choose the correct option.

A. 1 only

B. 1 and 3 only

C. 2 and 3 only

D. All of the above

Correct Answer: D

Solution All the statements are correct,

Kaziranga is located between latitudes 26°30' N and 26°45' N, and longitudes 93°08' E to 93°36' E within two districts in the Indian state of Assam -the Kaliabor subdivision of Nagaon district and the Bokakhat subdivision of Golaghat district. http://www.jigyasaias.com/Admin/printDailyTest/DTEST59C4E063D0E4D 2/6 9/25/2017 Print Question Paper

The park area is circumscribed by the , which forms the northern and eastern boundaries, and the Mora Diphlu, which forms the southern boundary. Other notable rivers within the park are the Diphlu and Mora Dhansiri.

Agoratoli Eco Tourism Resort is located in the lush green habitant of Kaziranga National Park.

Ref: http://www.thehindu.com/news/national/other-states/kaziranga-to-open-from-oct-2/article19733362.ece (http://www.thehindu.com/news/national/other-states/kaziranga-to-open-from-oct-2/article19733362.ece)

http://www.thehindu.com/news/national/other-states/kaziranga-to-open-from-oct-2/article19733362.ece (http://www.thehindu.com/news/national/other-states/kaziranga-to-open-from-oct-2/article19733362.ece)

5. India's National Disaster Response Force (NDRF) in October 2017 will be Conducting the first of its kind Disaster management Exercise with a Regional grouping.

The exercise aimed at strengthening disaster management in South Asia will be Conducted with which of the Following Organisation?

A. SAARC

B. ASEAN

C. BIMSTEC

D. East Asian Assosiation

Correct Answer: C

Solution The exercise, to be conducted between October 10-13 and acronymed as BIMSTEC DMEx-2017, is the first such exercise aimed at strengthening disaster management in South Asia.

: India's National Disaster Response Force (NDRF) is going to conduct a disaster management exercise next month in New Delhi with BIMSTEC countries, the Home Ministry said on Wednesday.

Ref: http://economictimes.indiatimes.com/news/politics-and-nation/india-to-conduct-bimstec-disaster- management-exercise/articleshow/60768001.cms (http://economictimes.indiatimes.com/news/politics-and- nation/india-to-conduct-bimstec-disaster-management-exercise/articleshow/60768001.cms)

6. Which of the following statements is/are correct?

1. Narmada River rises from Maikala range near Amarkantak in Madhya Pradesh and drains into the Arabian sea, becoming the largest west flowing river of the peninsular India. 2. The Narmada water Disputes Tribunal has allocated the Narmada water amongst the States of Chattisgarh, Uttar Pradesh, Madhya Pradesh, Maharashtra and Gujarat.

Choose the correct option?

A. 1 only

B. 2 only

C. Both 1 and 2

D. Neither 1 nor 2

Correct Answer: A

Solution Statement 1 is correct, Narmada is the largest west flowing river of the peninsular India. It rises from Maikala range near Amarkantak in Anuppur district of Madhya Pradesh, at an elevation of about 1057 m. The total length of the river is 1,312 km and for the first 1079 km it flows in Madhya Pradesh and thereafter forms the common boundary between Madhya http://www.jigyasaias.com/Admin/printDailyTest/DTEST59C4E063D0E4D 3/6 9/25/2017 Print Question Paper

Pradesh and Maharashtra for 35 km, and Maharashtra and Gujarat for 39 km. In Gujarat State it stretches for 159 km. Its important tributaries are the Burhner, the Banjar, the Sher, the Shakkar, the Dudhi, the Tawa , the Ganjal, the Kundi, the Goi and the Karjan which joins from left whereas the Hiran, the Tendoni, the Barna, the Kolar, the Man, the Uri, the Hatni and the Orsang joins from right. Narmada drains into the Arabian Sea through the Gulf of Khambhat.

Second Statements is incorrect,

The Narmada water Disputes Tribunal (http://india-wris.nrsc.gov.in/wrpinfo/index.php?title=File%3AVolume2_2.pdf) has allocated the Narmada water amongst the States as below.

Madhya Pradesh18.25 Million Acre Feet Gujarat 9.0 Million Acre Feet Rajasthan 0.5 Million Acre Feet Maharashtra 0.25 Million Acre Feet Total 28.0 Million Acre Feet

7. Udaan is a Special Industry Initiative for Jammu & Kashmir, which of the following statements is/are correct about it?

1. It is funded by Ministry of Home Affairs and implemented by National Skill Development Corporation (NSDC). 2. Its objectives include providing exposure to the graduates and post-graduates of Jammu and Kashmir to the best of corporate India 3. The Udaan programme is designed to encourage corporates to travel to J&K meet with the youth and hire aspiring youth in J&K.

Choose the correct option.

A. 1 and 2 only

B. 2 and 3 only

C. 3 only

D. All of the above

Correct Answer: D

Solution Udaan is a Special Industry Initiative for Jammu & Kashmir in the nature of the partnership between the corporates of India and Ministry of Home Affairs and implemented by National Skill Development Corporation. The programme aims to provide skills training and enhance the employability of unemployed youth of J&K. The Scheme covers graduates, post- graduates, and three-year engineering diploma holders. It has two objectives:

(i) To provide an exposure to the unemployed graduates to the best of Corporate India;

(ii) To provide Corporate India, an exposure to the rich talent pool available in the State.

The Scheme aims to cover 40,000 youth of J&K over a period of five years and Rs. 750 crore has been earmarked for implementation of the scheme over a period of five years to cover other incidental expenses such as travel cost, boarding and lodging, stipend and travel and medical insurance cost for the trainees and administration cost. Further corporates are eligible for partial reimbursement of training expense incurred for the candidates who have been offered jobs.

Ref: https://udaan.nsdcindia.org http://www.skilldevelopment.gov.in/udaan.html (https://udaan.nsdcindia.org)

8. Quantitative Easing is an unconventional monetary policy by the central government, it's most likely effect will be?

A. Lower interest rates and increase the money supply.

B. Lower interest rates and Reduce in the money supply.

C. Increase in Gross Domestice Produce

D. Increase in Foreign Exchange Reserve http://www.jigyasaias.com/Admin/printDailyTest/DTEST59C4E063D0E4D 4/6 9/25/2017 Print Question Paper

Correct Answer: A

Solution Quantitative easing is an unconventional monetary policy in which a central bank purchases government securities or other securities from the market in order to lower interest rates and increase the money supply. Quantitative easing increases the money supply by flooding financial institutions with capital in an effort to promote increased lending and liquidity. Quantitative easing is considered when short-term interest rates are at or approaching zero and does not involve the printing of new banknotes.

Option A is the most appropriate answer.

Although option c and d could be its outcome but option A is the Direct and most effective outcome.

Ref: http://www.thehindu.com/opinion/editorial/taper-timetable/article19729704.ece (http://www.thehindu.com/opinion/editorial/taper-timetable/article19729704.ece)

9. The 6th edition of VASTRA was recently Inaugurated in Sitapura Industrial Area, Jaipur, India. It is a:

A. Skill Development initiative

B. An International Textile and Apparel Fair

C. Global Meet of Scientists for advocating Environment-Friendly dyes and Textile

D. Brainstorming session for water conservation.

Correct Answer: B

Solution The 6th edition of VASTRA i.e. "VASTRA - An International Textile and Apparel Fair 2017" (VASTRA - 2017) is scheduled during 21st to 24th September, 2017 at Jaipur Exhibition & Convention Centre, Sitapura Industrial Area, Jaipur, India. Rajasthan State Industrial Development and Investment Corporation Ltd. (RIICO) is the organiser and Federation of Indian Chambers of Commerce and Industry (FICCI) is the co-organiser of the fair.

VASTRA, an all-encompassing trade fair and conference on Textiles and Apparel, presents a fusion of the finest and the latest in textile products – from fibre to fashion, services and technology. Crafted to create business opportunities, it aims at revitalizing existing business ties and forging of new business relations. It is the ideal platform to enter JVs, strategic alliances and partnerships worldwide, launch new products, as well as, harness new locations for setting up businesses in India. It is also a place for disseminating technology and interacting with R&D Labs and experts for solution in technology and latest trends.

Ref: http://pib.nic.in/newsite/PrintRelease.aspx?relid=170999 http://vastratex.com/aboutus.php (http://pib.nic.in/newsite/PrintRelease.aspx?relid=170999)

10. Which of the following systems is/are correct about the Eastern Ghats?

1. The Eastern Ghats are discontinuous and irregular and dissected by rivers draining into the Bay of Bengal. 2. The Eastern Ghats are higher in elevation than the Western Ghats. 3. The eastern coastal plains are broader and are an example of an emergent coast.

Choose the correct answer?

A. 1 and 2 only

B. 2 and 3 only

C. 1 and 3 only

http://www.jigyasaias.com/Admin/printDailyTest/DTEST59C4E063D0E4D 5/6 9/25/2017 Print Question Paper

D. All of the above

Correct Answer: C

Solution Statement 2 is incorrect, The Western Ghats are higher in alleviation than the Eastern Coasts.

The Deccan Plateau is bordered by the Western Ghats in the west, the Eastern Ghats in the east and the Satpura, Maikal range and Mahadeo hills in the north. The Western Ghats are locally known by different names such as Sahyadri in Maharashtra, Nilgiri hills in Karnataka and Tamil Nadu and Anaimalai hills and Cardamom hills in Kerala. The Western Ghats are comparatively higher in elevation and more continuous than the Eastern Ghats.

Ref: http://ncert.nic.in/ncerts/l/kegy102.pdf http://ncert.nic.in/ncerts/l/iess102.pdf (http://ncert.nic.in/ncerts/l/kegy102.pdf)

http://www.jigyasaias.com/Admin/printDailyTest/DTEST59C4E063D0E4D 6/6 9/25/2017 Print Question Paper

1. Which of the following statements is/are correct about Nitrogen Cycle?

1. The ability to fix nitrogen is naturally found in certain bacteria and archaea. 2. Micro-organisms break down the molecules in excretions and dead organisms into ammonia, which can be directly taken up by plants.

Choose the correct option.

A. 1 only

B. 2 only

C. Both 1 and 2

D. Neither 1 nor 2

Correct Answer: C

Solution The nitrogen molecule (N2) is quite inert. To break it apart so that its atoms can combine with other atoms requires the input of substantial amounts of energy.

Three processes are responsible for most of the nitrogen fixation in the biosphere:

atmospheric fixation by lightning industrial fixation biological fixation by certain microbes — alone or in a symbiotic relationship with some plants and animals

Biological Fixation

The ability to fix nitrogen is found only in certain bacteria and archaea.

Some live in a symbiotic relationship with plants of the legume family (e.g., soybeans, alfalfa). Some establish symbiotic relationships with plants other than legumes (e.g., alders). Some establish symbiotic relationships with animals, e.g., termites and "shipworms" (wood-eating bivalves). Some nitrogen-fixing bacteria live free in the soil. Nitrogen-fixing cyanobacteria are essential to maintaining the fertility of semi-aquatic environments like rice paddies. Biological nitrogen fixation requires a complex set of enzymes and a huge expenditure of ATP. Although the first stable product of the process is ammonia, this is quickly incorporated into protein and other organic nitrogen compounds.

The proteins made by plants enter and pass through food webs just as carbohydrates do. At each trophic level, their metabolism produces organic nitrogen compounds that return to the environment, chiefly in excretions. The final beneficiaries of these materials are microorganisms of decay. They break down the molecules in excretions and dead organisms into ammonia.

Ref: http://www.biology-pages.info/N/NitrogenCycle.html (http://www.biology-pages.info/N/NitrogenCycle.html)

2. Which of the following statements is/are correct about Legislative Powers of the Governor?

1. The governor is an integral part of the state Legislature. 2. Governor of the state is the Ex-officio chairman of Legislative Council 3. Deciding whether a bill is money bill or not is the discretionary power of the governor, and beyond the scope of Judicial Review.

Choose the correct option

A. 1 only

B. 2 and 3 only

C. All of the above http://www.jigyasaias.com/Admin/printDailyTest/DTEST59C6212F6C372 1/6 9/25/2017 Print Question Paper

D. None of the above

Correct Answer: A

Solution Statement 1 is correct, the governor is an integral part of the State legislature, which comprise of Governor, State Legislative Assembly, and State Legislative Council (if exist in state)

Statement 2 is incorrect, Governor does not act as the presiding officer of the State Legislative Council.

Statement 3 is incorrect, Deciding on the matter of money bill is the power of speaker of the Legislative Council.

Ref: M.Laxmikanth.

3. Ocean acidification is the ongoing decrease in the pH of the Earth's oceans. Which of the following statements is/are correct about it?

1. When carbon dioxide (CO2) is absorbed by seawater, chemical reactions occur that reduce seawater pH over an extended period of time. 2. Ocean Acidification reduces the amount of carbon dioxide in the atmosphere.

Which of the following is/are correct?

A. 1 only

B. 2 only

C. Both 1 and 2

D. Neither 1 nor 2

Correct Answer: C

Solution Both the statements are correct.

When carbon dioxide (CO2) is absorbed by seawater, chemical reactions occur that reduce seawater pH, carbonate ion concentration, and saturation states of biologically important calcium carbonate minerals. These chemical reactions are termed "ocean acidification" or "OA" for short. Calcium carbonate minerals are the building blocks for the skeletons and shells of many marine organisms. In areas where most life now congregates in the ocean, the seawater is supersaturated with respect to calcium carbonate minerals.

The ocean absorbs about a quarter of the CO2 we release into the atmosphere every year, this way it reduces the amount of CO in the atmosphere but causes damage to the sea environment. 2 Ref: https://www.pmel.noaa.gov/co2/story/What+is+Ocean+Acidification%3F (https://www.pmel.noaa.gov/co2/story/What+is+Ocean+Acidification%3F) https://www.pmel.noaa.gov/co2/story/Ocean+Acidification (https://www.pmel.noaa.gov/co2/story/Ocean+Acidification)

4. (Basic Question for Beginners) 'Jaffna' Peninsula is a part of which Country.

A. Myanmar

B. Sri Lanka

C. Bangladesh

D. Iran

Correct Answer: B http://www.jigyasaias.com/Admin/printDailyTest/DTEST59C6212F6C372 2/6 9/25/2017 Print Question Paper

Solution The Jaffna Peninsula is an area in Northern Province, Sri Lanka. It is home to the capital city of the province, Jaffna and comprises much of the former land mass of the medieval Jaffna kingdom.

Ref: http://www.newindianexpress.com/states/andhra-pradesh/2017/sep/22/andhra-pradesh-to-set-up-industrial-corridor- in-sri-lanka-mou-soon-1660881.html (http://www.newindianexpress.com/states/andhra-pradesh/2017/sep/22/andhra- pradesh-to-set-up-industrial-corridor-in-sri-lanka-mou-soon-1660881.html)

5. Which of the following statements is/are correct regarding Organisation of Islamic Cooperation (OIC)?

1. Neighbouring countries of Pakistan, Bangladesh, Maldives are its members. 2. India has statues of observer member in this Cooperation. 3. It has key bodies like the Islamic Summit, the Council of Foreign Ministers (CFM), the General Secretariat.

Choose the correct option

A. 1 and 2 only

B. 2 and 3 only

C. 1 and 3 only

D. All of the above

Correct Answer: C

Solution Statement 2 is incorrect, India does not have an observer status in this organisation.

The Organisation of Islamic Cooperation (OIC) is the second largest inter-governmental organization after the United Nations with a membership of 57 states spread over four continents. The Organization is the collective voice of the Muslim world. It endeavours to safeguard and protect the interests of the Muslim world in the spirit of promoting international peace and harmony among various people of the world.

India is not associated with this organisation.

Ref: http://www.oic-oci.org/page/?p_id=52&p_ref=26&lan=en (http://www.oic- oci.org/page/?p_id=179&p_ref=60&lan=en) http://www.oic-oci.org/page/?p_id=179&p_ref=60&lan=en

6. Which of the following statements is/are correct about Volcanism (Volcanic Activity):

1. It is an Exogenic Force causing erosional and depositional changes on the landforms. 2. It is a source of obtaining direct information of Earth's Interiors. 3. The lava that is released during volcanic eruptions on cooling develops into igneous rocks.

Choose the correct option.

A. 1 and 2 only

B. 2 and 3 only

C. 3 only

D. All of the above

Correct Answer: B

Solution http://www.jigyasaias.com/Admin/printDailyTest/DTEST59C6212F6C372 3/6 9/25/2017 Print Question Paper

Statements 2 and 3 are correct.

Statement 1 is incorrect, Volcano is endogenic force causing sudden change, (Exogenic forces are like Rivers, Winds etc.)

Ref: http://ncert.nic.in/ncerts/l/gess203.pdf http://ncert.nic.in/ncerts/l/kegy203.pdf (http://ncert.nic.in/ncerts/l/gess203.pdf)

7. Which of the following statements is/are correct about Atmospheric Aerosols?

1. Aerosols are minute particles suspended in the atmosphere 2. The atmospheric aerosol is emitted from both natural and anthropogenic sources. 3. The atmospheric aerosol can influence our health causing reduced lung function, increased respiratory symptoms, cardiovascular diseases, and so forth.

Choose the correct option.

A. 2 only

B. 2 and 3 only

C. 1 and 3 only

D. All of the above

Correct Answer: D

Solution All the statements are correct.

Aerosols are minute particles suspended in the atmosphere. When these particles are sufficiently large, we notice their presence as they scatter and absorb sunlight. Their scattering of sunlight can reduce visibility (haze) and redden sunrises and sunsets.

Aerosols interact both directly and indirectly with the Earth's radiation budget and climate. As a direct effect, the aerosols scatter sunlight directly back into space. As an indirect effect, aerosols in the lower atmosphere can modify the size of cloud particles, changing how the clouds reflect and absorb sunlight, thereby affecting the Earth's energy budget.

Aerosols also can act as sites for chemical reactions to take place (heterogeneous chemistry). The most significant of these reactions are those that lead to the destruction of stratospheric ozone. During winter in the polar regions, aerosols grow to form polar stratospheric clouds. The large surface areas of these cloud particles provide sites for chemical reactions to take place. These reactions lead to the formation of large amounts of reactive chlorine and, ultimately, to the destruction of ozone in the stratosphere. Evidence now exists that shows similar changes in stratospheric ozone concentrations occur after major volcanic eruptions, like Mt. Pinatubo in 1991, where tons of volcanic aerosols are blown into the atmosphere

Ref: https://www.nasa.gov/centers/langley/news/factsheets/Aerosols.html (https://www.nasa.gov/centers/langley/news/factsheets/Aerosols.html) https://aerosol.ees.ufl.edu/atmos_aerosol/section07.html (https://aerosol.ees.ufl.edu/atmos_aerosol/section07.html)

8. (Basic Question for Beginners)

While India got its independence from Britishers in 1947, Puducherry was still under colonial rule and got its independence only in 1954.

It was under the colonial rule of?

A. Germany

B. France

C. Portugal

D. Italy

http://www.jigyasaias.com/Admin/printDailyTest/DTEST59C6212F6C372 4/6 9/25/2017 Print Question Paper

Correct Answer: B

Solution In 1674, Pondicherry became a French colony of the French colonial empire. Together with Chandernagor (already French since 1673), Mahé (since 1721), Yanam (Yanaon) (since 1731), Karaikal (Karikal) (since 1739) and Masulipatam (1760), it formed the French colony of French India, under a single French governor in Pondicherry, although French rule over one or more of these enclaves was repeatedly interrupted by British occupations. The territories of French India were completely transferred to the Republic of India de facto on 1 November 1954, and de jure on 16 August 1962, when French India ceased to exist, becoming the present Indian constituent union territory of Pondicherry, still combining four coastal enclaves.

Ref: https://www.py.gov.in/knowpuducherry/history.html http://mea.gov.in/bilateral-documents.htm? dtl/5302/Treaty+establishing+De+Jure+Cession+of+French+Establishments+in+India (https://www.py.gov.in/knowpuducherry/history.html)

9. Which of the following statements is/are correct regarding Ozone Depletion?

1. Ozone Layer depletion is observed in stratospheric layer of Atmosphere. 2. Ozone Layer depletion affects both orientation and mobility rates in phytoplankton, resulting in reduced survival rates for the organisms.

Choose the correct option.

A. 1 only

B. 2 only

C. Both 1 and 2

D. Neither 1 nor 2

Correct Answer: C

Solution Both the statements are correct.

Solar ultraviolet radiation creates an ozone layer in the atmosphere which in turn completely absorbs the most energetic fraction of this radiation. This process both warms the air, creating the stratosphere between 15 and 50 km altitude, and protects the biological activities at the Earth's surface from this damaging radiation.

The growing emissions of synthetic chlorofluorocarbon molecules cause a significant diminution in the ozone content of the stratosphere, with the result that more solar ultraviolet-B radiation (290–320?nm wavelength) reaches the surface.

Phytoplankton form the foundation of aquatic food webs. Phytoplankton productivity is limited to the euphotic zone, the upper layer of the water column in which there is sufficient sunlight to support net productivity. Exposure to solar UVB radiation has been shown to affect both orientation and motility in phytoplankton, resulting in reduced survival rates for these organisms.

Ref: https://www.epa.gov/ozone-layer-protection/health-and-environmental-effects-ozone-layer-depletion https://www.ncbi.nlm.nih.gov/pmc/articles/PMC1609402/ (https://www.epa.gov/ozone-layer-protection/health- and-environmental-effects-ozone-layer-depletion)

10. Which of the following statements is/are correct about SATH, a program providing ‗Sustainable Action for Transforming Human capital.?

1. SATH aims to identify and build future ‗role model‘ states for health systems 2. States of Assam, Uttar Pradesh and Karnataka were chosen for the implementation of SATH programme. 3. It is a joint initiative of Ministry of Health and Ministry of Home Affairs.

http://www.jigyasaias.com/Admin/printDailyTest/DTEST59C6212F6C372 5/6 9/25/2017 Print Question Paper

Choose the correct option.

A. 2 only

B. 1 and 2 only

C. 1 and 3 only

D. All of the above

Correct Answer: B

Solution Statement 3 is incorrect, it is a programme of NITI Aayog, with the state governments, Furthering the agenda for cooperative federalism.

SATH aims to identify and build three future ‗role model‘ states for health systems. NITI will work in close collaboration with their state machinery to design a robust roadmap of intervention, develop a program governance structure, set up monitoring and tracking mechanisms, hand-held state institutions through the execution stage and provide support on a range of institutional measures to achieve the end objectives. The program will be implemented by NITI along with McKinsey & Company and IPE Global consortium, who were selected through a competitive bidding process.

three states: Assam, Uttar Pradesh and Karnataka were chosen based on objective assessment criteria affecting the potential for impact and likelihood of success

Ref: http://pib.nic.in/newsite/PrintRelease.aspx?relid=171040 http://pib.nic.in/newsite/PrintRelease.aspx?relid=171040 http://www.business-standard.com/article/government-press-release/niti-aayog-and-govt-of-assam-organizes-workshop-on- health-sector-117092201101_1.html (http://pib.nic.in/newsite/PrintRelease.aspx?relid=171040)

http://www.jigyasaias.com/Admin/printDailyTest/DTEST59C6212F6C372 6/6 9/25/2017 Print Question Paper

1. Which of the following statements is/are correct about United Nations General Assembly?

1. It comprisies of all the members of United Nations. 2. Each of the Member States in the Assembly has one vote only. 3. Its annual meetings took place at headquater of UN in Washington.

Choose the correct answer.

A. 1 only

B. 1 and 2 only

C. 2 and 3 only

D. All of the above

Correct Answer: B

Solution Statement 3 is incorrect; the General Assembly meetings took place in New York and not in Washington. Statement 1 and 2 are absolutely Correct,

Established in 1945 under the Charter of the United Nations, the General Assembly occupies a central position as the chief deliberative, policymaking and representative organ of the United Nations. Comprising all 193 Members of the United Nations, it provides a unique forum for multilateral discussion of the full spectrum of international issues covered by the Charter. It also plays a significant role in the process of standard-setting and the codification of international law.

Each of the 193 Member States in the Assembly has one vote. Votes taken on designated important issues— such as recommendations on peace and security, the election of Security Council and Economic and Social Council members, and budgetary questions—require a two-thirds majority of Member States, but other questions are decided by a simple majority.

Ref: http://www.un.org/en/ga/about/background.shtml (http://www.un.org/en/ga/about/background.shtml)

2. Which of the following statements is/are correct about Alzheimer's Disease?

1. It is characterized by s progressive decline in cognitive function. 2. It is a common form of dementia disturbing multiple brain functions, including memory, thinking, orientation, etc.

Choose the correct option.

A. 1 only

B. 2 only

C. Both 1 and 2

D. Neither 1 nor 2

Correct Answer: C

Solution Both the statements are absolutely correct.

Dementia is a syndrome characterized by the disturbance of multiple brain functions, including memory, thinking, orientation, comprehension, calculation, learning capacity, language, and judgment. Consciousness is not clouded. The impairments of cognitive function are commonly accompanied and occasionally preceded, by deterioration in emotional control, social behavior, or motivation.1,2 Alzheimer disease is the most common form of dementia and possibly contributes to 60?70% of cases. Other types of dementia include vascular dementia, dementia with Lewy bodies, and a group of diseases that contribute to frontotemporal dementia. The boundaries between subtypes are indistinct and mixed forms often co-exist.

http://www.jigyasaias.com/Admin/printDailyTest/DTEST59C8BF4214D00 1/5 9/25/2017 Print Question Paper

Ref: http://www.thehindu.com/todays-paper/tp-opinion/recognising-alzheimers/article19749524.ece (http://www.thehindu.com/todays-paper/tp-opinion/recognising-alzheimers/article19749524.ece) http://www.who.int/medicines/areas/priority_medicines/BP6_11Alzheimer.pdf (http://www.who.int/medicines/areas/priority_medicines/BP6_11Alzheimer.pdf)

3. Deen Dayal Upadhyaya Grameen Kaushalya Yojana is a policy of which ministry?

A. Ministry of Corporate Affairs

B. Ministry of Commerce and Industries

C. Ministry of Skill Development and Entrepreneurship

D. Ministry of Rural Development

Correct Answer: D

Solution The Ministry of Rural Development (MoRD) announced the Deen Dayal Upadhyaya Grameen Kaushalya Yojana (DDU- GKY) Antyodaya Diwas, on 25th September 2014. DDU-GKY is a part of the National Rural Livelihood Mission (NRLM), tasked with the dual objectives of adding diversity to the incomes of rural poor families and cater to the career aspirations of rural youth.

Ref: http://pib.nic.in/newsite/PrintRelease.aspx?relid=171070 (http://pib.nic.in/newsite/PrintRelease.aspx?relid=171070)

http://ddugky.gov.in/content/about-us-0 (http://ddugky.gov.in/content/about-us-0)

4. Which of the following statements is/are correct about the Essential Services Maintainance Act.

1. The Essential Services Maintenance Act was enacted by the Parliament to maintain delivery of ―certain essential services and the normal life of the community.‖ 2. Under this act, only the Central Government can impose ESMA on services and ensure its uninterrupted supply.

Choose the correct option.

A. 1 only

B. 2 only

C. Both 1 and 2

D. Neither 1 nor 2

Correct Answer: A

Solution Statement 2 is incorrect.

This act gives power to the state to declare any services as Essential under this Act.

Both the Union and state government has the power to impose ESMA.

Ref: http://www.livemint.com/Politics/xVn03dKt9Y6mcgvj0et1KL/What-is-Esma.html http://www.ptinews.com/news/9097041_Karna-govt-invokes-ESMA-against-garbage-contractors- .html (http://www.livemint.com/Politics/xVn03dKt9Y6mcgvj0et1KL/What-is-Esma.html)

5. Which of the following statements is/are correct?

http://www.jigyasaias.com/Admin/printDailyTest/DTEST59C8BF4214D00 2/5 9/25/2017 Print Question Paper

1. Article 15 of the constitution Prohibition of discrimination on grounds of religion, race, caste, sex or place of birth. 2. Article 17 of the constitution abolishes Untouchability in any form. 3. Article 16 prevents the State from making any provision for the reservation of appointments or posts in favor of any backward class of citizens.

Choose the correct option.

A. 1 and 2 only

B. 3 only

C. 2 only

D. All of the above

Correct Answer: A

Solution Statement 3 is incorrect,

Article 16 Equality of opportunity in matters of public employment. Article 16(4) clearly says Nothing in this article shall prevent the State from making any provision for the reservation of appointments or posts in favor of any backward class of citizens which, in the opinion of the State, is not adequately represented in the services under the State.

Statement 1 and 2 are absolutely correct.

Ref: M.Laxmikanth

6. Pralay Sahayam exercise was conducted recently on the banks of Hussain Sagar Lake aimed towards preparedness for jointly tackling an urban flooding scenario.

This exercise was conducted between.

A. India and Maldives

B. India and SAARC

C. Multiple agencies of India

D. India and BIMSTEC

Correct Answer: C

Solution A multi-agency exercise was conducted on the banks of Hussain Sagar Lake as the final event of ‗Pralay Sahayam‘ in Hyderabad. The event demonstrated efforts of all central and state agencies, National Disaster Relief Force (NDRF) and the Armed Forces towards jointly tackling an urban flooding scenario in Hyderabad.

Ref: http://pib.nic.in/newsite/PrintRelease.aspx?relid=171060

7. Which of the following is not a Main Organ of United Nations?

A. UN General Assembly

B. Security Council

C. International Court of Justice

D. International Criminal Court http://www.jigyasaias.com/Admin/printDailyTest/DTEST59C8BF4214D00 3/5 9/25/2017 Print Question Paper

Correct Answer: D

Solution UN has 6 main organs wiz.

General Assembly. Security Council. Economic and Social Council. UN Secretariat. International Court of Justice. Trusteeship Council

Ref: http://www.un.org/en/sections/about-un/main-organs/ (http://www.un.org/en/sections/about-un/main-organs/)

8. Which of the following Skill Development Initiative is/are correctly Matched with the Ministry organizing it?

1. Hunar Hat: Ministry of Minority Affairs 2. USTTAD: Ministry of Home Affairs 3. Bharat Ke Kaushalzaade: Ministry of Rural Development

Choose the correct option.

A. 1 and 2 only

B. 1 and 3 only

C. 2 and 3 only

D. All of the above

Correct Answer: B

Solution The scheme ―Upgrading the Skills and Training in Traditional Arts/Crafts for Development (USTTAD) was launched in May 2015 and following activities have been taken up to assist the artisans; it is under the Ministry of Minority Affairs.

Ref: http://pib.nic.in/newsite/PrintRelease.aspx?relid=154166 (http://pib.nic.in/newsite/PrintRelease.aspx?relid=154166)

1 and 3 are correctly matched.

Ref: http://pib.nic.in/newsite/PrintRelease.aspx?relid=171067 http://pib.nic.in/newsite/PrintRelease.aspx?relid=171070 (http://pib.nic.in/newsite/PrintRelease.aspx?relid=171067)

9. Which of the following statements is/are correct about Rice Crop?

1. Rice needs high temperature, high humidity, and rainfall. 2. It grows best in the alluvial clayey soil, which can retain water. 3. In the areas of less rainfall, it grows with the help of irrigation

Choose the correct option

A. 1 only

B. 2 and 3 only

C. All of the above

D. None of the Above

Correct Answer: C

http://www.jigyasaias.com/Admin/printDailyTest/DTEST59C8BF4214D00 4/5 9/25/2017 Print Question Paper

Solution All the statements are absolutely correct.

: Rice is the major food crop of the world. It is the staple diet of the tropical and sub-tropical regions. Rice needs high temperature, high humidity, and rainfall. It grows best in the alluvial clayey soil, which can retain water. China leads in the production of rice followed by India, Japan, Sri Lanka and Egypt. In favorable climatic conditions as in West Bengal and Bangladesh two to three crops are grown in a year.

Ref: http://ncert.nic.in/ncerts/l/hess404.pdf (http://ncert.nic.in/ncerts/l/hess404.pdf)

10. 'Bathukamma' is a:

A. Ancient Water Storage Technique

B. Floral Festival in a Southern State

C. Miniature Painting of North East region

D. Boat Race in a Southern

State Correct Answer: B

Solution Bathukamma is floral festival celebrated by the Hindu women of Telangana. Every year this festival is celebrated as per Shalivahana calendar for nine days starting Bhadrapada Amavasya till Durgashtami, usually in September– October of Gregorian calendar.

Ref: http://www.thehindu.com/news/national/telangana/let-the-streets-brighten-up-bathukamma-is-here/article19716853.ece (http://www.thehindu.com/news/national/telangana/let-the-streets-brighten-up-bathukamma-is-here/article19716853.ece)

http://www.jigyasaias.com/Admin/printDailyTest/DTEST59C8BF4214D00 5/5